You are on page 1of 258

1.

All of the following are present in


Superficial perineal pouch except:
a) Sphincter urethrae muscles

b) Bulbospongiosus

c) Posterior scrotal nerves

d) Duct of Bulbourethral glands

Correct Answer - A
A i.e. Sphincter urethral muscles
Root, crura & bulb of penis (i.e. 1 bulb/corpus spongiosa & 2
crura/corpora cavernosa), bulbospongiosus muscle, ducts of
bulburethral gland, and posterior scrotal nerve/artery4 are contents
of superficial perineal pouch.
Deep perineal pouch contains bulbourethral glands, membranous
urethra, sphincter urethrae muscle (internal urethral sphincter) and
dorsal nerve of peris

Join Telegram Channel @mmedicalbooks for all NEET


PG /JIPMER/PGI/FMGE /AIIMS Questions papers
withanswers and explainations and All kinds of
TEXTBOOKS & NOTES Free Available in the Channel t.me/
mmedicalbooks or click on red box for the channel link
2. Structure not passing through the
esophageal hiatus:
a) Left phrenic nerve

b) Right vagus nerve

c) Left vagus nerve

d) Left gastric artery

Correct Answer - A
Aortic opening lies in osteoaponeurotic part of diaphragm between
right & left crusQ at T1212 vertebral level and it transmits - aorta,
azygous vein & thoracic ductQ.
Oesophageal opening lies in muscular portion derived from right
crus at TIO levelQ and it transmits oesophagusQ, oesophageal
branch of left gastric arteryQ and both right & left vagus (gastric)
nervesQ
Right phrenic nerve passes through the central tendon of diaphragm
at T8 levelQ, either through venacaval aperture (with IVC)Q or just
lateral to it. Whereas, left phrenic nerve passes through the
muscular part of diaphragm anterior to central tendon, just lateral to
left cardiac surface and more anterior than the right phrenic nerve.

Join Telegram Channel @mmedicalbooks for all NEET PG /JIPMER/PGI/


FMGE /AIIMS Questions papers withanswers and explainations and All
kinds of TEXTBOOKS & NOTES Free Available in the Channel t.me/
mmedicalbooks or click on red box for the channel link
3. Dilator pupillae is supplied by
a) Postganglionic parasympathetic fibers from Edinger Westphal
nucleus

b) Post ganglionic sympathetic fibers from cervical sympathetic


chain

c) Turd CN

d) Sympathetic fibers from Vi nerve

Correct Answer - B
B i.e. Post ganglionic sympathetic fibers from cervical sympathetic
chain
Dilator iridis (pupillae) is supplied by postganglionic (nonmyelinated)
fibres from superior cervical sympathetic gangliaQ.
Parasympathetic Sympathetic (adrenergic)
(cholinergic pathway pathway to dilator
to sphincter pupillae) iridis (pupillae)
Nucleus Fibers start from 1st order neuron from
Edinger-Westphal hypothalamus till ciliospinal
nucleus near the (Budge) center in
rd
3 nerve nucleus in interomediolateral columns of
floor of aqueduct of Cs-T1
sylvius
Preganglionic Through occulomotor 2nd order preganglionic
neuron (3rd) nerve neuron from Budge ciliospinal
center tosuperior cervical
ganglion in neck (cervical
sympathetic neu)
Ganglia Ciliary ganglionQ Superior cervical ganglionQ
Post Short ciliary nerve 3rd order post ganglionic
ganglionic fibers arising from superior
neuron cervical ganglion,
ascends carotid
plexus (along internal carotid
artery) to enter skull, where
they join nasociliary branch of
trigeminal nerve.

Join Telegram Channel @mmedicalbooks for all NEET PG /


JIPMER/PGI/FMGE /AIIMS Questions papers withanswers and
explainations and All kinds of TEXTBOOKS & NOTES Free
Available in the Channel t.me/mmedicalbooks or click on red box
for the channel link
4. The first centres of ossification appears
during which month of pregnancy
a) At the end of 2nd month of pregnancy

b) At the beginning of 3rd month of pregnancy

c) At the end of 3rd month of pregnancy

d) At the end of 4th month of pregnancy

Correct Answer - A
A i.e. At the end of 2nd month of pregnancy
Clavicle is the first bone in the body to start ossifying and is ossified
from 3 centres. The shaft of clavicle is ossified from 2 (medical &
lateral) primary centres, which appear between the 5th & 6", weeks of
intra uterine life, and fuse about the 45th day meeting between the
middle and lateral third of clavicle. A secondary centre for sternal
end (and sometimes for acromial end) appears in late teens or early
twenties

Join Telegram Channel @mmedicalbooks for all NEET PG /JIPMER/PGI/


FMGE /AIIMS Questions papers withanswers and explainations and All
kinds of TEXTBOOKS & NOTES Free Available in the Channel t.me/
mmedicalbooks or click on red box for the channel link
5. Which of the following is not true about
glomerular capillaries')
a) The oncotic pressure of the fluid leaving the capillaries is less
than that of fluid entering it

b) Glucose concentration in the capillaries is the same as that in


glomerular filtrate

c) Constriction of afferent arteriole decreases the blood flow to the


glomerulas

d) Hematocrit of the fluid leaving the capillaries is less than that of


the fluid entering it

Correct Answer - A
Ans-a. The oncotic pressure of the fluid leaving the capillaries is less
than that of fluid entering it Guyton I LpJ1 4-.;anong 23/e p653-6_)
Glomerular oncotic pressure (due to plasma protein content) is
higher than that of filtrate oncotic pressure in Bowman's capsule
"Since glucose is freely filtered and the fluid in the Bowman's
capsule is isotonic with plasma, the concentration of glucose in the
filtrate is the same as in the capillaries

Join Telegram Channel @mmedicalbooks for all NEET PG /


JIPMER/PGI/FMGE /AIIMS Questions papers withanswers and
explainations and All kinds of TEXTBOOKS & NOTES Free
Available in the Channel t.me/mmedicalbooks or click on red
box for the channel link
6. Mean arterial pressure is calculated as:
a) (SBP + 2DBP)/ 3

b) (DBP + 2SBP)/ 3

c) (SBP + 3DBP)/ 2

d) (DBP + 3SBP)/ 2

Correct Answer - A
A i.e. (SBP + 2DBP) /3

Join Telegram Channel @mmedicalbooks for all NEET PG /


JIPMER/PGI/FMGE /AIIMS Questions papers withanswers and
explainations and All kinds of TEXTBOOKS & NOTES Free
Available in the Channel t.me/mmedicalbooks or click on red box
for the channel link
7. Adverse effects of hypothermia are all
except:
a) Cardiac arrhythmias

b) Renal failure

c) Decreased peripheral resistance

d) Reversible coagulopathy

Correct Answer - C
C i.e. Decreased peripheral resistance
Adverse effects of hypothermia are:
Bradycardia
Hypotension
Reduced cardiac output
Reduced stroke volume
Reduced blood flow and hyperviscosity
Microembolism
Coagulation is prolonged
Hypoxic injury to the liver.
8. Who discovered the structure and amino
acid sequence of insulin as well as the
interspecies differences in molecular
structure and won the Nobel prize for?
a) Prof. JJ Abel

b) Frederick Sanger

c) Rutter, Goodman and Uhlich

d) Frederick Banting and Charles Best

Correct Answer - B
Ans: b. Frederick Sanger /Ref Lehninger 5/e p903-904, 292)
Frederick Sanger discovered the structure and amino acid sequence
of insulin as well as the interspecies differences in molecular
structure and got the Nobel Prize for that.
Fredrick Sanger again with Walter Gil,bert got Nobel prize in
1980Jim sequencing of large nucleic acid (DNA) molecule."

Join Telegram Channel @mmedicalbooks for all NEET PG /


JIPMER/PGI/FMGE /AIIMS Questions papers withanswers
and explainations and All kinds of TEXTBOOKS & NOTES Free
Available in the Channel t.me/mmedicalbooks or click on red
box for the channel link
9. Adrenaline, noradrenaline and dopamine
act through
a) Single pass receptors

b) Four pass receptors

c) Seven pass receptor

d) Ligand gated channel

Correct Answer - C
C i.e. Seven pass - receptor
Catecholamines (epinephrine, norepinephrine and dopamine) act
through G protein coupled receptors (GPCRs), which span the cell
membrane 7 times; hence also referred to as seven pass (seven
helix) or serpentine receptors.
• Catecholamines
(adrenaline, noradrenaline
& dopamine) act through G
protein
coupled receptors. al

adrenargic
catecholamine use
phospholipid C(IP3 ?

DAG) and az &
adrenergic
catecholamines use C-AMP

(adenyl
cyclase) second

messenger system.
• Hetrotrimeric GTP
binding proteins (G?
proteins) are cell

membrane proteins that
have seven trans

membrane segments that
loop (pass) in & out of cell

membraneQ.
(serpentine protein)
• Cytoplasmic tail is
coupled to GTP that has
3 part (trimeric) - a, 13, y

subunits.

Join Telegram Channel @mmedicalbooks for all NEET PG /


JIPMER/PGI/FMGE /AIIMS Questions papers withanswers and
explainations and All kinds of TEXTBOOKS & NOTES Free
Available in the Channel t.me/mmedicalbooks or click on red
box for the channel link
10. Administration of pure O2 to hypoxic
patients is dangerous because:
a) Apnea occurs due to hypostimulation of peripheral chemo
receptor

b) Pulmonary edema

c) DPG

d) Convulsions

Correct Answer - A
A i.e. Apnea occurs due to hypostimulation of peripheral chemo
receptor

Join Telegram Channel @mmedicalbooks for all NEET


PG /JIPMER/PGI/FMGE /AIIMS Questions papers
withanswers and explainations and All kinds of
TEXTBOOKS & NOTES Free Available in the Channel
t.me/mmedicalbooks or click on red box for the
channel link
11. Which of the following is not the primary
function of gut flora?
a) Synthesis of short chain fatty acids

b) Decreased proliferation of epithelial cells

c) Production of vit. K

d) Fermentation of mucin

Correct Answer - B
B i.e. Decreased proliferation of epithelial cells
Gut (colonic) microflora synthesize short chain fatty acids (SCFAs)
by fermenting complex carbohydrate, resistant starches and other
components of dietary fibers & mucin that escape digestion in upper
git. They also produce vitamin K and biotin, detoxify xenobiotics,
increase & maturate immunity, , preventing allergy and growth of
pathogenic bacteria. They promote development of normal colonic
epithelium and stimulate intestinal epithelial cell differentiation (ie
increase not decrease proliferation of epithelial cells)
Fermentation & Digestion of Complex Carbohydrate
- Colonic bacteria metabolize meal components that are not
digested by host enzymes and make their products available to the
body via fermentation. It provides advantage to the host in that the
colonic bacteria are capable of performing metabolic reactions that
do not take place in mammalian cells.
-Formation of Short Chain Fatty Acids (SCFAs)- Colonic bacteria
form SCFAs by femanting carbohydrates (starches, fibers). SCFAs
are 2-5 carbon weak acids with 80mmol/L average normal
concentration in lumen. About 60% of this is acetate, 25%
propionate and 15% butyrate.- SCFAs are absorbed & metabolized
to make a significant contribution to total calory intake - butyrate is a
critical energy source for colonocytes.
- SCFAs excert a trophic effect on colonic epithelium cells. The
colonic epithelium turns over rapidly even in health,
12. All the statements regarding LDL
receptors are true except:
a) Present only in the extra-hepatic tissues

b) Clathrin-coated pits contain the receptor on cell membrane

c) It is taken into the cells by endocytosis

d) Increased cellular cholesterol down-regulates the synthesis of


LDL receptors

Correct Answer - A
A i.e. Present only in the extra-hepatic tissues:
13. True about isoenzymes is:
a) Same quaternary structure

b) Same distribution in different organs

c) Same enzyme classification with same umbers

d) Catalyze the same reaction

Correct Answer - D
Ans: D
* Isoenzymes are the multiple forms of the same enzyme in a single
species that catalyze the same chemical reaction or reactions, but
differ from each other structurally, electrophoretically and
immunologically.
* Though the same chemical reaction is catalyzed, the different
isoenzymes may catalyze the same reaction at different rates.
* Isoenzymes have different pH optimes, Km and V max values.
* Isoenzymes may differ in their amino acid sequence and
their quarternary structures.
* The isoenzymes may have different properties also for e.g. LDH-4
and LDH-5 are easily destroyed by heat, whereas LDH-1 and LDH-2
are not, if heated upto 60°C. (Heat resistant).
* Individual isoenzymes (isozymes) are distinguished and numbered
on the basis of electrophoretic mobility, with the number 1 being
assigned to that form having the highest mobility toward the anode,
for e.g. LDH-1 has the highest mobility towards the anode and LDH-
5 is the slowest.
* Isoenzymes have different tissue distributions. Therefore the
pattern of isoenzymes found in the plasma may serve as a means of
identifying the site of tissue damage. Example of the diagnostic use
of isoenzymes are the study of Lactate Dehydrogenase and
Creatine Kinase.
14. All of the following are true regarding
oxygenases except :
a) Incorporate 2 atoms of oxygen

b) Incorporate 1 atom of oxygen

c) Required for hydroxylation of steroids

d) Required for carboxylation of drugs

Correct Answer - D
Ans. d. Required for carboxylation reaction (Ref Harper 2R/e p9R-
102: Lehninger 5/e p8l 6).
Oxygenase (which belongs to oxido-reductase class) cause
incorporation of 2 atoms of 0, (dioxygenase) or 1 atom of 02
(monoxygenase or hydroxylase). Carboxylase enzyme, which
belongs to ligase class is required for carboxylation.
Cytochromes P450 are (heme containing)
monoxygenases° which take part in hydroxylase cycle° (NADH,
NADPH and Fe2 S2 required), detoxify drugs in liver
microsomes° (with cytochrome b5, Fe2 S2 not required).
15. Increased levels of alanine in serum after
fasting suggest:
a) Increased release of alanine from muscle

b) Reduced amino acid utilization for gluconeogenesis

c) Break in continuity of plasma membrane resulting in leakage of


amino acids

d) Decreased uptake of alanine by liver

Correct Answer - A
A i.e. Increased release of alanine from muscle:lanong 24/e p2 I
* except for the first few hours of starvation, the major effects are
progressive depletion of tissue fat and protein.
* Fat depletion is steady and continues unabated until most of the fat
stores in the body are gone.
* Proteins undergo three phases of depletion-rapid depletion of
mobilizable proteins at first, then greatly slowed depletion (during
this time increased fat breakdown leads to a state of ketosis).
* this finally, rapid depletion again shortly before death. The major
aim of protein catabolism during a state of starvation is to provide
the glucogenic amino acids (especially alanine and glutamine) that
serve as substrates for gluconeogenesis in the liver."
16. Chymotrypsinogen is a
a) Zymogen

b) Carboxypeptidase

c) Transaminase

d) Elastase

Correct Answer - A
A i.e. Zymogen
Chymotrypsinogen or prochymotrypsin is a proprotein/ proenzyme /
or zymogen.
Proproteins (Proenzymes or Zymogens)
* Zymogen is an inactive precursor of enzyme / protein that require
selective proteolysis for activation. Selective proteolysis involves one
or more highly specific proteolytic clips that may or may not be
accompanied by separation of resulting peptide. So selective
proteolysis often result in conformational changes that create the
catalytic site of an enzyme or it unmasks the active site of an
enzyme by removing small region of peptide chain (by hydrolysis of
specific peptide bond).
* Enzymes needed intermittently but rapidly often are secreted in an
initially inactive form since new synthesis and secretion of required
proteins might be insufficiently rapid to respond to a pressing
pathophysiological demand such as clot formation, clot dissolution,
tissue repair etc. So proenzymes facilitate rapid mobilization of
activity in response to physiological need.
Proteolytic digestive enzymes are known as proteases. These are
secreted as inactive zymogens. The synthesis of enzymes as
Zymogens protects the cell from being digested by its own
product (autodigestion). Proteases are of two types
17. If a 4 nucleotides sequence code for an
amino acid instead of 3, then theoretically
how many unique amino acids could be
coded by such a system?
a) 16

b) 64

c) 128

d) 256

Correct Answer - D
Ans. d. 256 Ref. Harper 28/e p353
Number of Nucleotides in Number of Coded Amino
Codon Acids
2 42=16
3 43= 64
4 44=256
Three is the minimum number of nucleotides per codon needed to
encode 20 amino acids.
a. 20 amino acids are encoded by combinations of 4 nucleotides
b. If a codon were two nucleotides, the set of all combinations could
encode only
4x4 = 16 amino acids.
c. With three nucleotides, the set of all combinations can encode
4x4x4 = 64 amino acids (i.e. 64 different combinations of four
nucleotides taken three at a time).
d. With three nucleotides,
4x4x4x4 = 256 amino acids
18. If more than one codon codes for same amino acid, this phenomenon is known
as?

a) Degeneracy

b) Frameshift mutation

c) Transcription

d) Mutation

Correct Answer - A
3 out of 64 codons do not code for any amino acid. These have been termed nonsense
codons.
These nonsense codons are utilized in the cell as termination signals; they specify where
the polymerization of amino acids into a protein molecule is to stop.
The remaining 61 codons code for 20 amino acids.
Thus, there must be degeneracy in the genetic code, ie; multiple codons must decode the
same amino acid.

Some amino acids are encoded by several codons; for example six different codons, UCU,
UCC, UCA, UCG, AGU, and AGC all specify serine. Other amino acids, such as methionine
and tryptophan, have a single codon. In general, the third nucleotide in a codon is less
important than the first two in determining the specific amino acid to be incorporated, and
this accounts for most of the degeneracy of the code.

However, for any specific codon, only a single amino acid is indicated; with rare exceptions,
the genetic code is unambiguous—that is, given a specific codon, only a single amino acid
is indicated. The distinction between ambiguity and degeneracy is an important
concept.

Ref: Weil P. (2011). Chapter 37. Protein Synthesis & the Genetic Code. In D.A. Bender,
K.M. Botham, P.A. Weil, P.J. Kennelly, R.K. Murray, V.W. Rodwell (Eds), Harper's
Illustrated Biochemistry, 29e.
19. Poly(A) tail translates into (i.e. on
translation give rise to):
a) Polyproline

b) Polylysine

c) Polyalanine

d) Polyglycine

Correct Answer - B
B i.e. Polylysine
Codon AAA GGG CCC TTT/ UUU
Translation
LysinQ Glycine Proline Phenylalanine
product
20. CAP in LAC operon is
a) >Positive regulator

b) >Negative regulator

c) >Attenuation

d) >Constitutive expression

Correct Answer - A
Ans. A. >Positive regulator
Positive regulator Ref Lippincott's biochemistry 4/e chapter 32 (3/e
p418-420)
What is an Operon?
* In prokaryotes, the genes coding for proteins involved in a
particular metabolic pathway are often sequentially arranged
together on the chromosome along with a single promoter or
regulatory region. This entire cluster is termed as an operon, for eg,
the lac operon (coding proteins for metabolism of lactose) or trp
operon (coding proteins needed for the synthesis of tryptophan).
* An operon is regulated by the single promoter or regulatory region.
All the genes of an Operon are transcribed together as a unit. The
transcription product of operon is a single polycistronic messenger
RNA (mRNA). Thus all the genes of an operon are controlled
together, that is, they all are turned on or off as a unit.
When both glucose and lactose are available:
* In this case, transcription of the lac operon does not take place,
even if lactose is present at a high concentration.
* Presence of glucose deactivates Adenylyl cyclase—so no cAMP–
CAP complex forms and the CAP-binding site remains empty. RNA
polymerase is, therefore, unable to effectively initiate transcription,
even though the repressor may not be bound to the operator region.
Consequently, the three structural genes are not expressed.
* (Lac Operon regulation is very beautifully depicted in a figure of
Lippincott biochemistry; Fig. 32.4 of 4/e and Fig. 30.11 of 3/e)
21. Starting material for production of insulin
from bacteria is:
a) Genomic DNA from beta pancreatic cells of human

b) Genomic DNA from lymphocytes of human

c) mRNA from beta pancreatic cells of human

d) mRNA from lymphocytes of human

Correct Answer - C
C i.e. mRNA from beta pancreatic cells of human
Starting material for production of insulin from bacterial cells (E.coli)
using recombination DNA technology is mRNA from beta pancreatic
cells of humanQ.
22. Which of the following is not used in
polymerase chain reaction?
a) DNA Polymerase

b) Taq polymerase

c) Dideoxyribonucleotides

d) DNA template

Correct Answer - C
PCR is an in vitro DNA amplification procedure in which millions of
copies of a particular sequence of DNA can be produced within a
few hours.
The reaction cycle has the following steps:
Step 1: Separation (Denaturation): DNA strands are separated
(melted) by heating at 95°C for 15 seconds to 2 minutes.
Step 2: Priming (Annealing): The primers are annealed by cooling to
50°C for 0.5 to 2 minutes. The primers hybridize with their
complementary single-stranded DNA produced in the first step.
Step 3: Polymerization: New DNA strands are synthesized by Taq
polymerase. This enzyme is derived from bacteria Thermus
aquaticus that are found in hot springs.
The steps of 1,2 and 3 are repeated. In each cycle, the DNA strands
are doubled. Thus, 20 cycles provide for 1 million times
amplifications. These cycles are generally repeated by
automated instrument, called Tempcycler.
5. After the amplification procedure, DNA hybridization technique or
Southern blot analysis with a suitable probe shows the presence of
the DNA in the sample tissue.
Dideoxyribonucleotides are not used in the polymerase chain
reaction.
23. The 40 nm gap in between the
tropocollagen molecule in collagen which
serve as the site of bone formation is
occupied by which of the following-
a) Carbohydrate

b) Ligand moiety

c) Calcium

d) All

Correct Answer - C
C i.e. Calcium
40 nm gap between the typrocollagen molecues serve as nucleation
site for deposition of hydroxyapatite I ca5(p04)3(19H),1 with some
phosphate] crystals in boneQ.
24. If you are in PHC, which anticoagulant is
used to sent the blood sample for blood
glucose estimation?
a) EDTA

b) Heparin

c) Potassium oxalate

d) Potassium oxalate + sodium fluoride

Correct Answer - D
Ans. is 'd' i.e., Potassium oxalate and sodium fluoride
* Sodium fluoride- potassium oxalate mixture is used for glucose
determination.
* NaF inhibits the glycolytic enzymes responsible for the breakdown
of glucose in the blood ( At room temperature, about 10% of the
glucose is lost per hour from an untreated sample).
* The potassium oxalate is the primary anticoagulant, as NaF has a
poor anticoagulant effect.
25. In a well fed state, the activity of CPT-I in
outer mitochondrial membrane is
inhibited by:
a) Glucose

b) Pyruvate

c) Acetyl CoA

d) Malonyl CoA

Correct Answer - D
D i.e. Malonyl CoA
In well fed state, the activity of CPTI (carnitine palmitoyl transferase
I) in outer mitochondrial membrane is inhibited by melonyl CoAQ.
26. Which of the following is not required for
protein synthesis of eukaryotes:
a) RNA polymerase

b) Ribosomes

c) Peptidyl transferase

d) Amino acyl tRNA synthetase

Correct Answer - A
Ans. a. RNA polymerase (Ref Harper 28/e p362) RNA polymerase
enzyme is involved in transcription process, not in translation.
The a-amino group of the new aminoacyl-tRNA in the A site carries
out a nucleophilic attach on the esterified carboxyl group of the
peptidyl-tRNA occupying the P site (peptidyl or polypeptide site).
Peptidyl transferase: Catalyses two reactions, peptide bond
formation between amino acids and together with release factor,
peptide release.
27. During prolonged starvation, the rate of
gluconeogenesis depends on
a) Increased alanine levels in liver

b) Decreased cGMP in liver

c) ADP in liver

d) Decreased essential fatty acids in liver

Correct Answer - A
A i.e. Increased alanine level in muscle
28. Technique (s) used to detect Gene
Mutation is/are
a) RT-PCR

b) Denaturing gradient gel electrophoresis

c) DNA sequencing and Restriction fragment polymorphism


(RFLP) both

d) All

Correct Answer - D
ALL i.e. (RT-PCR, Denaturing gradient gel electrophoresis, DNA
sequencing, Restriction fragment polymorphism (RFLP), Single-
strand conformational polymorphism
29. Subtelomeric rearrangement of genes is frequently associated with mental
retardation. All of the following techniques can be used to diagnose them,
EXCEPT:

a) FISH

b) MAPH

c) CGHarray

d) MALDI

Correct Answer - D
MALDI: It is a technique used to assess the molecule, based on their mass only.
Most commonly employed methods for dispersing peptides, proteins, and other large
biomolecules into the vapor phase for mass spectrometric analysis are electrospray
ionization and matrix-assisted laser desorption and ionization, aka MALDI.

In electrospray ionization, the molecules to be analyzed are dissolved in a volatile solvent
and introduced into the sample chamber in a minute stream through a capillary .
As the droplet of liquid emerges into the sample chamber, the solvent rapidly disperses
leaving the macromolecule suspended in the gaseous phase. The charged probe serves to
ionize the sample.
Electrospray ionization is frequently used to analyze peptides and proteins as they elute
from an HPLC or other chromatography column already dissolved in a volatile solvent.

In MALDI, the sample is mixed with a liquid matrix containing a light-absorbing dye and a
source of protons.
In the sample chamber, the mixture is excited using a laser, causing the surrounding matrix
to disperse into the vapor phase so rapidly as to avoid heating embedded peptides or
proteins.

Ref: Kennelly P.J., Rodwell V.W. (2011). Chapter 4. Proteins: Determination of Primary
Structure. In D.A. Bender, K.M. Botham, P.A. Weil, P.J. Kennelly, R.K. Murray, V.W.
Rodwell (Eds), Harper's Illustrated Biochemistry, 29e.
30. An abnormal Ham test is most likely
associated with which of the following ?
a) Defect in spectrin

b) Defective GPI anchor

c) Defect in complement

d) Mannose-binding residue defect

Correct Answer - B
Ans. is 'b' i.e., Defective GPI anchor
* Ham test is used for PNH, and you all know that in PNH there is
deficiency of GPI-linked proteins (GPI anchor).
31. The most common gene defect in
idiopathic steroid resistant nephrotic
syndrome -
a) ACE

b) NPHS 2

c) HOX II

d) PAX

Correct Answer - B
Ans. is 'b' i.e. NPHS-2 Ref: Lewis J.B., Neilson E.G. (2012). Chapter
283. Glomerular Diseases. In D.L. Longo, A.S. Fauci, D.L. Kasper,
S.L. Hauser, J.L. Jameson, J. Loscalzo (Eds), Harrison's Principles
of Internal Medicine, 18e.
Genetic basis of proteinuria in nephrotic syndrome
* Recently certain gene mutations have been recognized which are
associated with certain glomerulonephritis, producing nephrotic
syndrome.
* The gene mutations codes certain proteins and the common
feature of these proteins is their localization to the structures of the
glomerular filtration barrier, such as slit diaphragm and podocyte
cytoskeletal structures such as actin.
* Their specific functions and interaction are incompletely
understood, but it is clear that the integrity of each is necessary to
maintain the normal glomerular filtration barrier.
Gene Chromosome ProteinLocation Disease
NPHS slipt diaphragm Nephrotic
19(413 nephrin
1 syndrome of finnish type
NPHS2 slit diaphragm Steroid
podocin
podocin
1x25-31 resistant nephrotic syndrome
32. A person with radiologically confirmed
reflux nephropathy develops nephrotic
range proteinuria. Which of the following
would be the most likely histological
finding in the patient?
a) Focal segmental glomerulosclerosis

b) Nodular glomerulosclerosis

c) Membranous glomerulopathy

d) Proliferative glomerulonephritis with crescents

Correct Answer - A
Ans. is 'a' i.e., Focal segmental glomerulosclerosis
o Reflux nephropathy causes focal segmental glomerulosclerosis
(FSGS).
33. The anticoagulant of choice for
performing coagulation studies is -
a) EDTA

b) Heparin

c) Tri sodium citrate

d) Double oxalate

Correct Answer - C
Ans. is 'c' i.e., Tri sodium citrate, 3.2% trisodium citrate (Ref. Ronald
Hoffman Hem, ,rogv/513, mtrobes 12/c 3.2% trisodium citrate is the
anticoagulant of choice for coagulation test
The most commonly use anticoagulant for in vitro Coagulation
Studies is Citric Acid.It is generally employed in the form of Sodium
Citrate.
In Vitro use of anticoagulants
i) For coagulation studies-Sodium Citrate
ii) For Hematological studies-EDTA
iii) For estimation of ESR
1. Wintrobes method-Double oxalate.
2. Westergren method-Sodium Citrate.
34. In acute inflammation due to the
contraction of endothelial cell
cytoskeleton, which of the following
results -
a) Delayed transient increase in permeability

b) Early transient increase in permeability

c) Delayed permanent increase in permeability

d) Early permanent increase in permeability

Correct Answer - B
Ans. is 'b' i.e., Early transient increase in permeability
Increased vascular permeability
* The hallmark of acute inflammation is increased vascular
permeability
The following mechanisms have ben proposed for increased
permeability.
i) Formation of endothelial gaps in venules (immediate transient
response)
* This is the most common mechanism of vascular leakage and is
caused due to the mediators such as histamine, bradykinin,
leukotrienes, neuropeptide sustance
* Classically this type of leakage affects venules, leaving capillaries
and arterioles unaffected.
* The precise reason for this restriction to venules is uncertain, it
may be because there is greater density of receptors for mediators
in venular endothelium.
* Binding of mediators such as histamine to their receptors on
endothelial cells activate intracellular signaling pathways that lead
to phosphorylation of contractile and cytoskeletal proteins such as
myosin.
* These proteins contract leading to contraction of endothelial cells
and separation of intercellularjunction.
* Thus the gaps in the vascular endothelium are largely intercellular
or close to the intercellular junctions.
* This type of leakage occurs rapidly after exposure to the mediator
and is usually reversible and short lived (15-30 minutes), it is thus
known as immediate transient response.
* Cytokines such as interleukin-1 (IL-1), tumour necrosis factor
(TNF) and interferon y also increase vascular permeability by
inducing a structural reorganization of the cytoskeleton such that
the endothelial cells retract from one another.
* In contrast to the histamine effect, the cytokine induced response
is somewhat delayed (4-6 hrs) and long lived (24 hrs or more).
ii) Direct endothelial injury resulting in endothelial cell necrosis
and detachment (immediate sustained response)
* This effect is usually encountered in necrotizing injuries and is due
to direct damage to the endothelium by injurious stimulus e.g severe
burns or lytic infections.
* The reaction is known as immediate sustained response because
the leakage starts immediately after the injury and is sustained at
high levels for several hours until the damaged vessels are
thrombosed or repaired. o All levels of microcirculation are affected
including venules, capillaries and arterioles.
iii) Delayed prolonged leakage
* This is curious but relatively common type of increased
permeability that begins after a delay of 2-12 hours, lasts for several
hours or even days and involves venules as well as capillaries.
* This type of leakage is caused by mild to moderate thermal injury,
X-ray radiation or ultraviolet radiation and certain bacterial toxins.
* It is caused either by direct effect of injurious agent or by cytokine
mediated endothelial retraction.
iv) Leucocyte mediated endothelial injury
* Leucocytes adhere to endothelium relatively early in inflammation.
* These leucocytes may be activated releasing toxic oxygen species
and proteolytic enzymes which then cause endothelial injury or
detachment.
* This type of leakage affects venules (mostly); pulmonary &
glomerular capillaries.
v) Increased transcytosis across the endothelial cytoplasm
* Transcytosis occurs across channels consisting of clusters of
interconnected uncoated vesicles and vacuoles called the
vesiculovacuolar organelle, many of which are located close to
intercellular junctions.
* It usually occurs in venules.
vi) Leakage from new blood vessels
* During repair formation of new vessels occur (angiogenesis)
* New vessel sprouts, remain leaky until the endothelial cells mature.
35. In hemodialysis associated amyloidosis,
which of the following is seen
a) Transthyretin

b) Beta-2 microglobulin

c) SAA

d) Alpha microglobulin

Correct Answer - B
Ans. is 'b' i.e., B2 microglobulin
Dialysis-related amyloidosis (DRA) is a disabling disease
characterized by accumulation and tissue deposition
of amyloid fibrils consisting of beta2-microglobulin (beta2-m) in the
bone, periarticular structures, and viscera of patients with chronic
kidney disease (CKD)
36. True regarding prothrombin time
measurement?
a) Platelet rich plasma is required

b) Activate with kaolin

c) Should be measured within 2 hours

d) Immediate refrigeration to preserve coagulation factor viability

Correct Answer - C
Ans. is 'c' i.e., Should be measured within 2 hours
Major screening tests of coagulation
* In all of these tests, blood is collected with a calcium- chelating
anticoagulant ( usually citrate) and then centrifuged to remove the
erythrocytes, leukocyte, and platelets.
* The remaining platelet-poor plasma (PPP) serves as the substrate
for these tests.
* Depending upon the test, the phospholipid is added to the PPP to
substitute for platelets and an activating agent is added.
* The PPP is then recalcified and the time needed for a clot to form
is measured.
The sample to be kept at room temperature if it is to be used for
prothrombin time, Lupus anticoagulant or factor VII assays and to be
kept at 4° C for other assays.
Once blood is drawn for testing, the tests should be completed
within a time frame of 30 minutes to 2 hours
Measurement of prothrombin time:
* The PT measures the function of the extrinsic and common
coagulation pathways. It is commonly used to monitor oral
anticoagulant therapy.
* Method:- A source of phospholipid and tissue thromboplastin (
rabbit or human brain extract) is added to platelet-poor plasma, and
Ca2+ is added to initiate coagulation. The time for the appearance of
fibrin strands (PT) is measured and is compared with the control.
* Normal PT is 11-13 seconds.
37. A 10 year old child presents with pallor &
history of blood transfusion 2 months
back. On investigation, Hb -4.5gm, total
count 60000, platelet count- 2lakhs and
CD 10(+) ve, CD 19 (+)ve, CD 117 (+) ve,
MPO (+) ve & CD 33(-)ve. What is the most
likely diagnosis?
a) ALL

b) AML

c) Undifferentiated leukemia

d) Mixed phenotypic acute leukemia

Correct Answer - D
Ans. is 'd' i.e., Mixed phenotypic acute leukemia
* Immunological markers in this question are :-
* CD 10 (+) ye, CD 19(+) ye --> B cell lineage
Myeloperoxidase (MPO) ( +)ve, CD117 (+)ve
Myeloid lineage
* Immunological markers of two lineage are present. Therefore,it is
a case of biphenotypic leukemia (mixed phenotypic leukemia).
* Biphenotypic leukemia is a subtype of "leukemia of ambiguous
lineage" (a new class of AML which has been added in 18th/e of
Harrison. But in Harrison, detailed description of this class has not
been given.. Acute leukemia with Ambiguous lineage
3 Human acute leukemias are broadly classified as myeloid or
lymphoid according to the expression of surface and/or cytoplasmic
antigens.
* Uncommonly, the lineage of origin is not clear ; either two
separate blast populations are encountered, one myeloid and other
lymphoid, or a single blast population demonstrating evidence of
both myeloid and lymphoid differentiation concurrently.
* Acute leukemia with ambiguous lineage has following subtypes :-.
A) Acute undifferentiated leukemia
In this leukemia, the blast cells express none of the antigen that are
useful in lineage attribution but rather express only markers that are
linked to high stages of immaturity such as CD 34, CD 38 and HLA-
DR and at times CD 7.
B) Mixed phenotypic acute leukemia
In this type of leukemia, immunological marker of more than one
lineage are present. This is classifed into​i) Bilinear acute leukemia
* In this, two separate blast populations are encountered.
* For example blast cells of lymphoid and myeloid lineage are
present simultaneously.
n) Biphenotypic acute leukemia
* Bipheotypic acute leukemia is also called " hybrid acute
leukemia" or " acute mixed lineage leukemia" or "simultaneous
leukemia'.
* In this leukemia, a single blast population demonstrates marker of
more than one lineage (in contrast to bilinear leukemia where blasts
of two different lineage are present).
* Coexpression of myeloid with B lymphoid lineage is most common
type (65%). Co-presence of myeloid with T-lymphoid lineage is
second most common type. Co-expression of T and B cells
phenotype or trilineage phenotype (T, B and myeloid cells) is rare.
* According to the 2008 WHO classification, the requirements for
lineage attribution are as follows :?
a) Myeloid lineage :- Presence of myeloperoxidase or at least 2
markers of monocytic differentiation CD11c, CD 14, CD64,
lysozyme, non-specific esterase
b) T lineage :- Cytoplasmic or surface CD3
c) B lineage :- Strong CD19 expression with at least one of the
following weakly expressed CD 10, CD 79a, cytoplasmic CD22; or
weak CD 19 expression with at least 2 of the following strongly
expressed CD 10, CD 79a, cytoplasmic CD22.
Clinical presentation
Clinical presentation
* Similar to other acute leukemias, acute leukemia with ambiguous
lineage presents with symptoms related to depression of marrow
function, i.e., cytopenia i.e. anemia, leukopenia, thrombocytopenia
or pancytopenia.
Coming back to question
* Expression of B lineage (CD19 with CD 10) and myeloid lineage (
myeloperoxidase) confirms the diagnosis of biphenotypic leukemia.
38. A 22-year-old female gives the history of
recurrent joint pains. She has now
developed petechial hemorrhages. She is
most likely to have:
a) Megakaryocytic thrombocytopenia

b) Amegakaryocytic thrombocytopenia

c) Platelet function defects

d) Acquired factor VIII inhibitors

Correct Answer - D
Answer:. d. Acquired factor VIII inhibitors (Wintrobes p1442-
144-.'74;Harrison19/ep734-735, 740, 18/e p982).
The clinical presentation in a young female of recurrent joint pains
with petechial hemorrhage is suggestive of an autoimmune disease.
A female patient is unlike to have hemophilia, as it is an X-linked
disorder. However, she can have autoantibodies against factor VIII.
39. Which of the following is true for
octreotide?
a) Stimulates growth hormone secretion

b) Used in secretory diarrhea

c) Orally active

d) Contraindicated in acromegaly

Correct Answer - B
Ans. b. Used in secretory diarrhoea (Ref Goodman Gilman 12/e
p1338-1339. 1346: Katling 11/e p650)
Octreotide is used in secretory diarrhea
Octreotide is an octapeptide analog of somatostatin, effective in
inhibiting the severe secretory diarrhea brought about by hormone-
secreting tumours of the pancreas and the GI tract."
40. Peripheral conversion of T4 to T3 is
inhibited by?
a) Propranolol

b) Diltiazem

c) Sotalol

d) Sodium iodide

Correct Answer - A
Ans. is 'a' i.e., Propranolol
41. All of the following drugs are correctly
matched with their metabolism except:
a) Digoxin: p-glycoprotein

b) Simvastatin: glucuronidation

c) Verapamil: CYP3A4

d) Carvedilol: CYP2D6

Correct Answer - A
ANSWER: a. Digoxin: p-glycoprotein (Ref Goodman Gilman 12/e
759, 1346; Kat:-g p58-60i
* P-glycoprotein is not involved in the metabolism of digoxin.
* "The cytochrome P450 (CYP) isoform CYP3A4 serves as the
major pathway for metabolism of lovastatin, simvastatinand
atorvastatin.
* Inhibition of activity of CYP3A4 can increase serum levels of these
statins.
* Glucuronidation is now considered a major pathway for elimination
of the active metabolites of statins.
* The glucuronidation is carried out by UGT1A1 and UGT1A3
enzymes.
42. Brinzolamide is a
a) Highly specific irreversible and noncompetitive carbonic
anhydrase inhibitor

b) Highly specific irreversible and competitive carbonic anhydrase


inhibitor.

c) Highly specific reversible and competitive carbonic anhydrose


inhibitor

d) Highly specific reversible and non-competitive carbonic


anhydrase inhibitor.

Correct Answer - D
Ans is 'd' i.e. Highly specific reversible and noncompetitive
carbonic anhydrase inhibitor
* Carbonic anhydrase inhibitors act by a non-competitive,
reversible inhibition of the enzyme carbonic anhydrase.
* Brinzolamide is a highly .specific inhibitor of carbonic anhydrase II
(CA II), which is the main carbonic anhydrase isoenzyme involved in
the secretion of aqueous humour.
* Inhibition caused by brinzolamide is reversible and non-
competitive.
43. A 35-year-old male is put on thiazides for
the treatment of primary hypertension.
What would be the status of his urinary
sodium, potassium and calcium in the
first 24-hours of drug administration?
a) Sodium and potassium increases, calcium increases

b) Sodium and potassium decreases, calcium decreases

c) Sodium and calcium increases, potassium decreases

d) Potassium and calcium increases sodium decreases

Correct Answer - A
Answer: a. Sodium and potassium increases, calcium increases (1(e
j..Ciuodnian unman iLe pod,
* Urinary sodium, potassium and calcium increase in the first 24
hours of thiazide administration.
* The acute effects of thiazide on Ca" excretion is variable:
* It has been seen in various trials that urinary excretion increases in
the initial stages of thiazide diuretic therapy. When administered
chronically, thiazide diuretics decrease Ca" excretion.
44. Which of the following drugs can be used
as a transcranial patch for treatment of
parkinsonism?
a) Levodepa

b) Rotigotine

c) Apomorphine

d) Aprantine

Correct Answer - B
Ans 'b' i.e. Rotigotine
* Rotigotine is intended to be delivered through transdermal
patches, so as to ensure a slow and constant dosage in a 24-hour
period.
45. Bremelanotide is used for -
a) Erectile dysfunction

b) LUTS

c) Hormone resistant prostate cancer

d) Metastatic RCC

Correct Answer - A
Ans is 'a' i.e. Erectile dysfunction
* Bremelanotide is an analogue of naturally occurring peptide alpha-
melanocyte-stimulating hormone (alpha​MSH).
* It stimulates erection in men and male rats, and was in clinical
trials for the treatments of erectile dysfunction in men and sexual
arousal disorder in women.
46. Lid retraction is caused by?
a) Apraclonidine

b) Brimonidine

c) Latanoprost

d) Travoprost

Correct Answer - A
Ans is 'a' i.e. Apracloridine
Apraclonidine may cause lid retraction in less than 1% of people.
Ocular side effects of topical agents for POAG
* β-blocker: Allergic blepharoconjunctivitis, corneal hypoesthesia,
blurred vision, dry eye, superficial punctate keratitis.
* Cholinomietics (pilocarpine): Blurred vision, miosis,
accommodative spasm, browache.
* Sympathomimetics
i) Non -selective (Dipivelrine): Follicular conjunctivitis, rebound
congestion, macular edema in aphakic
ii) Apraclonidine: Allergies, lid retraction, follicular conjunctivitis,
fluctuation in visual acuity
iii) Brimonidine: Ocular allergy, conjunctival blanching.
* Carbonic anhydrase inhibitors (Dorzolamide, brinaolamide):
Punctate keratitis, ocular allergies.
* Prostaglandin analogues (Latanoprost): Punctate keratitis, iris
pigmentation.
47. A patient who underwent abdominal
surgery, developed infection at wound
site. Awound swab was sent for culture,
which revealed MRSA. All of following
antibiotics can be used for his treatment
except -
a) Cefclor

b) Cotrimoxzate

c) Ciprofloxacin

d) Vancomycin

Correct Answer - A
Ans. is 'a' i.e., Cefclor
* Ciprofloxacin, Cotrimoxazole and vancomycin are used for MRSA
infections.
48. Chloroquine related eye disease risk is
a) HIV protease

b) HIV reverse transcriptase

c) HIV integrase

d) HIV entry into the cell

Correct Answer - B
Answer: . B. HIV reverse transcriptase >2 years (Ref Goodman and
Gilman 12/e p14051
Chloroquine related eye disease risk is high if duration of treatment
is more than 5 years
High Risk for Chloroquine
Age >60 years° High body fat°
Duration of treatment >5 years° Liver disease°
Daily dose >6.5 mg/kg/day of Renal disease°
3.0 mg/kg/day of Concomitant retinal disease°
49. The drug efavirenz inhibits?
a) HIV 1 Protease

b) HIV 1 reverse transcriptase

c) HIV1 Integrase

d) HIV entry into cell

Correct Answer - B
Ans is 'b' i.e. HIV1 reverse transcriptase
Efavirenz is a non-nucleoside reverse transcriptase inhibitor.
50. The most recent oral direct thrombin
inhibitor (DTI) for acute stroke prevention
is:
a) Ximelagatran

b) Idraparinux

c) Dabigatran

d) Wasfarean

Correct Answer - C
Ans. c. Dabigatran (Ref Katzung 11 /e p594)
The most recent oral direct thrombin inhibitor (DTI) for acute stroke
prevention is Dabigatran.
"In August 2011, Dabigatran was also licensed for prevention of
stroke and systemic embolism in patients with atrial fibrillation by the
European authorities, after approval for this indication was received
in October 2010 in USA.-
51.
Under section 304A of IPC, for a proved case
of medical negligence, The maximum
punishment of imprisonment is up to:
a) 1 year

b) 2 years

c) 3 years

d) 5 years

Correct Answer - B
Answer: b. 2 years (Ref Reddy 33/e p35, 29/e p260,
Under section 304A of IPC, for a proved case of medical negligence,
the maximum punishment of imprisonment is up to 2 years.
52. Giving false evidence under oath is
defined under section:
a) 151 IPC

b) 161 IPC

c) 181 IPC

d) 193 IPC

Correct Answer - D
D i.e. 193 IPC
Sec IPC 193 defines punishment for fabricating false evidence.
It is divided into two parts, first one lays done punishment for
intentional giving of false evidence in a judicial proceeding or
fabricating false evidence for the purpose of its being used in a
judicial proceeding.
The second one prescribes the punishment for intentionally giving
false evidence of fabricating false evidence in any case other than a
judicial proceeding.
Sec 191 IPC Defines what amounts to giving false evidence by a
person who is under legal obligation to speak the truth.
Sec 192 IPC Defines the offence of fabricating false evidence.
Sec 197 IPC Makes issuing or signing false certificates punishable.
Ref: Textbook on the Indian Penal Code By Krishna Deo Gaur 4th
edn page 332.
53. Which of the following IPC match is
Incorrect?
a) Section 300: Murder

b) Section 304: Culpable homicide not amounting to murder

c) Section 306: Attempt to suicide

d) Section 307: Attempt to murder

Correct Answer - C
Answer: c. Section 306: Attempt to suicide (Ref Reddy 33/e p292,
28/e /423-426; Textbook on the Indian Penal Code
* sec 309 IPC: Attempt to commit suicide -- ( 1 years imprisonment +
fine)
* When one attempts to commit suicide and fails it, that is
punishable under the Indian Penal Code. It means when a person
fails to commit suicide because of any reason only then Section 309
of Indian Penal Code will apply.
* shall be punished with simple imprisonment for a term which may
extend to one year or with fine, or with both.
54. For DNA testing, which tissue sample is
taken in an autopsy
a) Liver

b) Spleen

c) Kidney

d) Brain

Correct Answer - B
B i.e. Spleen
DNA can be extracted from any body fluid or tissue in which
nucleated cells are present. The samples for DNA testing in forensic
practice are usually taken from 1) Blood stains, 2) Semen, 3) Hair, 4)
tissues such as bone marrow, spleen, muscle, finger nail
scrapingsQ, 5) Mouth swabs, 6) Saliva stains, and 7) Chorionic villus
and muscle samples in foetus. Deep muscles, cardiac muscle,
bones (preferably long bones with more bone marrow) &, teeth
(preferably molar) are resistant samples and can be used in burnt &
decomposed bodies. Blood and spleen are easily obtainable
samples.
55. Flaying is seen in which type of
laceration:
a) Tear

b) Avulsion

c) Split

d) Stretch

Correct Answer - B
B i.e. Avulsion
- Tearing of skin & sub cutaneous tissue produced d/t blunt forceQ is
called laceration. In it the hair follicles and blood vessels are
crushedQ so external haemorrhage is not pronounced.
- On Scalp, Eyebrows, Cheek bones (zygoma)(2, Lower jaw, Iliac
crest, Perineum & ShinQ a lacerated wound looks like incised
wound.
- Avulsion or shearing laceration is produced by sufficient shearing
force delivered at a acute angle to tear off (detach) a portion of a
traumatized surface or viscus from its attachments. Flaying is a type
of avulsionQ in which shearing &grinding force by weight (such as of
lorry wheel passing over a limb) may produce avulsion (separation
of skin from underlying tissue) / degloving over a large area.
56. Palatoprints for identification of the
person is performed by taking prints from
which area of the hard plate
a) Lateral

b) Anterior

c) Middle

d) Posterior

Correct Answer - B
B i.e. Anterior
Superimposition technique determines whether the skull is that of a
person in photograph or notQ
Palatoprints for identification of persons is performed by taking prints
from anterior part of hard palateQ Cheiloscopy is utilization of lip
prints as a means of personal identificationQ
57. Stack formula is for age estimation from
dental pattern in:
a) Infants

b) Adults

c) 25-50 years age group

d) Above 50

Correct Answer - A
A i.e. Infants
58. When a doctor issues a false medical
certificate, then he is liable under:
AIIMS 11; Bihar 11; NEET 15
a) Sec. 197 IPC

b) Sec. 87 IPC

c) Sec. 304A IPC

d) Sec. 338 IPC

Correct Answer - A
Ans. Sec. 197 IPC
59. An alcoholic patient is brought for
medical examination for alleged assault of
his neighbor. ON examination, abnormal
bhaviour with irrelevant talk is noted. He
has history of abstaining from alcohol for
past 4 days and is diagnosed to be a case
of delirium tremens. What is true
according to the Indian Penal Code
a) He is riminnaly responsible for his act

b) He is partially responsible for his act

c) Have diminished responsibility of his act

d) Not responsible for his act under section 84, IPC

Correct Answer - D
D i.e. Not responsible for his act under section 84, IPC
Mc Naughten Rules are framed for the criminal responsibility of the
insaneQ. And it states that an accused person is not legally
responsible for crime if he/she is not of a sound mindQ. It is
accepted in India as law of criminal responsibility in section 84
IPCQ.
In Mc Naughten rule, insanity is a legal (not psychiatric) concept and
it judges delusion as if realQ.
Drunkeness (intoxication) is never an excuse against criminal
responsibility unless it becomes a disease (such as alcohol
paranoia, delirium tremens). And then an act done by a person of
unsound mind, who is uncapable of knowing the nature of the act
due to intoxication is not an offence (section 84 IPC)
due to intoxication is not an offence (section 84 IPC)
60. According to the transplantation of
Human organs act 1994, what is the
punishment for a doctor found guilty
a) Less than I year

b) Less than 2 year

c) 2 to 5 years

d) More than 5 years

Correct Answer - C
C ie 2-5 years
If a doctor is found guilty for unauthorized removal of human organ,
or for commercial dealings etc, he can be punished with
imprisonment up to 5 years and fine of 10000 Rs and removal of
name from register of state medical council for 2 year for first
offence and permanently for subsequent offence (according to the
transplantation of human organs act 1994). But a/t 2011 Amendment
the punishment has become imprisonment upto 10 years with fine
extending upto 20 lakh Rs and removal of name from register of
state medical council for a period of 3 years for first offence and
permanently for subsequent offence.
61. An elderly male presented with fever, dry
cough and chest pain. The causative
organism was detected in sputum which
is cultured on charcoal yeast medium. The
organism is:
a) H. influenzae

b) Moraxella catarrhalis

c) Legionella

d) Burkholderia cepacia

Correct Answer - C
This is a case of Legionella infection. The established approach to
diagnosis combines direct fluorescent antibody (DFA) with culture of
infected tissues.
Cultures must be made on buffered charcoal yeast extract
(BCYE) agar medium that includes supplements (amino acids,
vitamins, L-cysteine, ferric pyrophosphate), which meets the
growth requirements of Legionella.

It is buffered to meet the acidic conditions—optimal for Legionella


growth (pH 6.9).
The isolation of large Gram-negative rods on BCYE after 2 to 5 days
that have failed to grow on routine media (blood agar, chocolate
agar) is presumptive evidence for Legionella.
Diagnosis is confirmed by DFA staining of bacterial smears prepared
from the colonies.
Ref: Ray C.G., Ryan K.J. (2010). Chapter 34. Legionella. In C.G.
Ray, K.J. Ryan (Eds), Sherris Medical Microbiology, 5e.
62. Which is NOT TRUE with regards to
antibiotic resistance in bacteria?
a) Most common mechanism is production of antibiotic destroying
enzymes

b) Lack of target binding site may be mechanism of resistance in


some enterococci against vancomycin

c) Plasmid mediated resistance is transmitted only vertically

d) Target binding shifting is the mechanism in some pneumococci

Correct Answer - C
Plasmid mediated resistance can be transmitted both horizontally
and vertically. Routes of horizontal transfer of plasmid mediated
resistance are conjugation, transduction and transformation.

3 main mechanisms of drug resistance:


Production of beta lactamases: Most common mechanism of
penicillin resistance in both gram +ve and -ve bacteria.
Alteration of PBP (penicillin binding proteins ): Seen in gram +ve
bacteria. This is the basis for methicillin resistance in MRSA also
basis for penicillin resistance in pneumococci and enterococci.
Impaired penetration: Seen only in gram -ve bacteria
PBP are localised on outer surface of cell membrane and not on cell
wall. PBP are essential for synthesis of cell wall.
63. With reference to Bacteroides fragilis the
following statements are true, EXCEPT:
a) B. fragilis is the same frequent anaerobe isolated from clinical
samples.

b) B. fragilis is not uniformly sensitive to metronidazole

c) The lipopolysaccharide formed by B. fragilis is structurally and


functionally different from the conventional endotoxin

d) Shock and disseminated intravascular coagulation are common


in Bacteroides bacteremia

Correct Answer - D
Ans. D. Shock and disseminated intravascular coagulation are
common in Bacteroides bacteremia
Bacteroides species have lipopolysaccharides, but lack the
lipopolysaccharide structures with endotoxic activity (including beta-
hydroxymyristic acid).
The lipopolysaccharides of B fragilis are much less toxic than those
of other gram-negative bacteria.
Thus, infection caused by Bacteroides does not directly
produce the clinical signs of sepsis (eg, fever and shock) so
important in infections caused by other gram-negative bacteria.
When these clinical signs appear in Bacteroides infection, they are a
result of the inflammatory immune response to the infection.
Ref: Brooks G.F. (2013). Chapter 21. Infections Caused by
Anaerobic Bacteria. In G.F. Brooks (Ed), Jawetz, Melnick, &
Adelberg's Medical Microbiology, 26e.
64. A young male patient presented to OPD
with urethral discharge. On urine
examination pus cells were found but not
organisms. Which method would be the
best for culture
a) McCoy cell line

b) Thayer Martin medium

c) PPLO broth

d) Cooked meat medium

Correct Answer - A
Ans. a. McCoy cell line
Urine examination in a patient of urethral discharge demonstrating
only pus cells but no gram-negative diplococcic suggest "non-
gonococcal urethritis". The most common cause of non-gonococcal
urethritis is Chlamydia trachomatis. Chlamydia trachomatis is
cultured on McCoy cells, HeLa cells or Hep-2 cells.
Culture Media Organism
McCoy cell line ChlamydiaQ
Thayer Martin
NeisserieQ
medium
PPLO broth MycoplasamQ
Cooked meat Cooked meat
medium mediumQ
Organisms causing Non-gonococcal Urethritis
Chlamydia Gardnerella
trachomatis (MC)
Q vaginalisQ
Urea plasma
Candida albicans
urealyticumQ
Mycoplasma Trichomonas
genitaliumQ vaginalis
Mechanical or
Herpes virusQ
chemical iinitation
CMVQ
65. Bacteria coated with complement and Ig;
phagocytosis is enhanced by-
a) Receptor mediated endocytosis

b) Pseudopod formation

c) Myeloperoxidase mediated destruction

d) C3b - Fc mediated destruction

Correct Answer - D
Ans. is 'd' i.e., C3b- Fc mediated destruction
Here, the examiner is simply asking about the opsonization.
Complement (especially C3b) and Ig coating the bacteria act as
opsonin and enhance phagocytosis.
The speed of phagocytosis can be increased markedly by bringing
into action two attachment devices present on the surface of
phagocytic cells
i) Fe receptor :- which binds the Fc portion of antibody
molecule, chiefly IgG
ii) Complement receptor (CR3) :- which binds the third
component of complement especially C3b.
This coating of the organisms by molecules that speed up
phagocytosis, is termed `opsonization' and the Fc portion of antibody
and C3b are termed as opsonins.
Now the question crops, how does opsonins enhance phagocytosis :
?
Both the membrane of a phagocytosing cell and its target have a
negative charge (zeta-potential), making it difficult for the two cells to
come close together. Once the opsonins attach to the target, the
negative charged is masked. Take note that the negative charge of
the target doesn't disappear. The opsonin simply overrides the
charge, making it easier for white blood cells (phagocytic cells), to
undergo phagocytosis. During the process of opsonization (also,
opsonisation), antigens are bound by antibody or complement
molecules. Phagocytic cells express receptors, CR1 and Fc
receptors, that bind opsonin molecules, C3b and antibody,
respectively. With the antigen coated in these molecules, binding of
the antigen to the phagocyte is greatly enhanced. In fact, most
phagocytic binding cannot occur without opsonization of the antigen.
Furthermore, opsonization of the antigen and subsequent binding to
an activated phagocyte will cause increased expression of
complement receptors on neighboring phagocytes.
About option c
As you all know, inside the phagocytes, bacteria are destroyed by
lysosomal enzymes (lactoferrin, lysozyme
defensins) and reactive oxygen intermediate (myeloperoxidase
system, superoxide, H202, hydroxy radicals).
However, this process has nothing to do with opsonins. Opsonins
enhance phagocytosis by masking the negative charge on the
bacterium so that it easily comes in contact with phagoctes.
66. A 5-year-old child from a rural area
presented to the OPD with pustular
lesions on the lower legs. The cuture from
the lesion showed hemolytic colonies on
the blood agar which were Gram-positive
cocci. Which of the following reactions
would help to provisionally confirm the
diagnosis of group A streptococcal
pyoderma?
a) Optochin sensitivity

b) Bacitracin sensitivity

c) Catalase positivity

d) Bile solubility

Correct Answer - B
Answer: b. Bacitracin sensitivity (Ref Ananthanaravan 8/e pe p205-
206)
Gram-positive cocci with alpha hemolytic colonies on sheep agar are
Streptococcus viridians and Streptococcus pneumoniae.
They can be further differentiated on basis of optochin sensitivity,
Streptococcus viridians-optochin resistantor Streptococcus
pneumonia-optochin sensitive.
67. A patient admitted to an ICU is on central
venous line for the last one week. He is on
ceftazidime and amikacin. After 7 days of
antibiotics he develops a spike of fever
and his blood culture is positive for gram
positive cocci in chains, which are
catalase negative. Following this,
vancomycin was started but the culture
remained positive for the same organism
even after 2 weeks of therapy. The most
likely organism causing infection is-
a) Staphylococcus aureus

b) Viridans streptococci

c) Enterococcus faecalis

d) Coagulase negative Staphylococcus

Correct Answer - C
Ans. is 'c' i.e., Enterococcus fecalis
Option 'a' & 'cl' can be excluded by following information in question
: ?
1) The isolated cocci are arranged in chains while staphylococci
are arranged in grape like clusters.
2) The isolated organism is catalase negative while staphylococci
are catalase positive.
Now two options are left :?
Option 'b' - Viridans streptococci
Option 'b' - Viridans streptococci
Option 'c' - Enterococcus fecalis
These two can be differentiated by following points :-
1)Enterococci are a frequant cause of nosocomial bacteremia in
patients with intravascular catheters, while viridans streptococcal
bacteremia occurs in neutropenic patients, particularly after bone
marrow transplantation or high dose chemotherapy for cancer.
2)Enterococci are intrinsically resistant to all cephalosporins and
also show high level of resistance to aminoglycosides. Resistance to
vancomycin has also become quite common with enterococci, while
viridans streptococci are sensitive to vancomycin and vancomycin is
the drug of choice for presumptive treatment of viridans grp
bacteremia.
- Now it is obvious that the infecting organism in this patient is
enterococcus as patient has developed bacteremia on central
venous line and he is not responding to cephalosporine,
aminoglycoside and vancomycin.
68. First chemical barrier encountered by
microorganism for common exposed sites
-
a) Lysozyme

b) Acidic pH

c) Skin

d) Lactose

Correct Answer - B
Ans. is 'b' i.e., Acidic pH
Human body can defend itself against infection-causing organisms
by the use of passive barriers to prevent micro​organisms entering
inside the body.
All surfaces of the body, which come in contact with the external
environment, are equipped with mechanical and chemical barriers to
prevent penetration by microorganisms.
The skin is essentially an impermeable covering.
Surface secretions constitute one of the most important mechanisms
of innate immunity
69. Biological indicator for determining
efficacy of autoclaving is:
September 2009
a) Pseudomonas aeruginosa

b) Clostridium Perfringenes

c) Bacillus stearothermophilus

d) Salmonella typhi

Correct Answer - C
Ans. C: Bacillus stearothermophilus
70. Best epidemiological study is -
a) RCT

b) Meta-analysis

c) Cohort study

d) Case-control study

Correct Answer - B
Ans. is 'b' i.e., Meta-analysis
Here are the different epidemiological studies with decreasing order
of accuracy to test the association between risk factor and disease
:?
1. Systematic review and meta-analysis → Overall most reliable
2. Randomized controlled trials (controlled clinical trails) → Most
reliable individual study.
3. Retrospective (Non-concurrent/historic) Cohort study.
4. Prospective (concurrent) Cohort study.
5. Case control study
6. Cross-sectional study
7. Ecological study
71. Founder members of vision 2020 are A/E
a) WHO

b) UNICEF

c) ORBIS

d) International Agency for prevention of blindness

Correct Answer - B
B i.e. UNICEF
72. Epiphora is :
a) Cerebrospinal fluid running from the nose after fracture of
anterior cranial fossa

b) An epiphenomenors of a cerebral tumor

c) An abnormal overflow of tears due to obstruction of lacrimal


duct

d) Eversion of lower eyelid following injury

Correct Answer - C
C i.e. Abnormal overflow of tears due to obstruction of lacrimal duct
73. Which of the following does not affect the
Hardy Weinberg principle?
a) Small population

b) Random mating

c) Mutations

d) Gene outflow

Correct Answer - B
Ans. is 'b' i.e., Random mating
Hardy-Weinbergt ,aw of genetic equilibrium
The ilardyAN einberg law plays an important role in the field
ofpopulation genetics and often serves as a basis for genetic
inference.
o Hardy-Weinberg equation refers to a state in which the allele and
the genotype frequencies do not change from one generation to the
next in a population.
Five conditions are required in order for a population to remain at
Hardy-Weinberg equilibrium:
1.A large breeding population
2. Random mating
3. No change in allelic frequency due to mutation
4. No immigration or emigration
5.No natural selection
So, random mating maintains equilibrium (does not disturb or affect).
Non-random mating inluence it.
74. Regarding Evidence Based Medicine
(EBM), all of the ing arc true, except:
a) EBM depends on clinical model and decision analysis to base
its recommendations

b) EBM objectively evaluates the quality of clinical research by


critical assessing techniques reported by researchers in
publications

c) The strongest argument for the therapeutic intervention is


systemic review or meta-analysis of triple blinded, randomized,
placebo-controlled clinical trials with allocation concealment and
complete follow-up involving a homogenous population of
patients or medical disorders

d) The weakest argument is the opinion of experts or medical


boards

Correct Answer - A
Answer: a. EBM depends on clinical model and decision analysis to
base its recommendations
It aims to apply the best available evidence gained from the scientific
method to medical decision-making
and it seeks to assess the quality for evidence of the risks and
benefits of treatments.
EBM is the conscientious, explicit and judicious use of current best
evidence in making decisions about the care of individual patients
75. You have diagnosed a patient clinically as
having SLE and ordered 6 tests. Out of
which 4 tests have come positive and 2
are negative. To determine the probability
of SLE at this point, you need to know-
a) Prior probability of SLE; sensitivity and specificity of each test

b) Incidence of SLE and predictive value of each test

c) Incidence and prevalence of SLE

d) Relative risk of SLE in this patient

Correct Answer - A
Ans. is 'a' i.e. Prior probability of SLE; sensitivity and specificity of
each test
* Probability allows for an accurate method of calculating how much
the likelihood of changes as new information (i.e., a test result)
becomes available.
* Probability of disease after a given test (post-term probability) can
be calculated by (Bayes' theorem)
Post test probability = Pretest probability x Likelihood ratio
Pre test probability
* It is the probability of disease before getting a test.
* There is a principal "The interpretation of new information depends
on old information".
* That means the interpretation of a test result (post-test probability)
depends on the probability of disease before test (pre-test
probability).
* Pre-test probability depends on prevalence, infact prevalence rate
is described as pre-test probability of having a disease.
Likelihood ratio
* The likelihood ratio provides a direct estimate of how much a test
result will change the probability of having the disease.
* Tests results are often expressed as either positive or negative.
* So, there is
A likelihood ratio for a positive test result (LR +), and
A likelihood ratio for a negative test result (LR-)
* The positive likelihood ratio (LR +) tells us how much to increased
the probability of a disease if the test is positive. It is calculated by
following formula.
The likelihood ratio is generally a better descriptor than
sensitivity or specificity because it more directly describes the effect
of a test result on the probability of disease and it incorporates both
the sensitivity and specificity.
So, in short
* To know the probability of a disease after a test (post test
probability) depends upon :
i) Pretest probability —> Prevalence
ii) Likelihood ratio —> Specificity & sensitivity
76. Which of the following is not a feature of a
cross sectional study -
a) Not expensive

b) Observational study

c) Used for chronic diseases

d) Well suited for establishing interrelationships of diseases

Correct Answer - D
Ans. is 'd' i.e., Well suited for establishing interrelationship of
diseases
Why it is not possible to establish a cause and effect relationship ?
Lets see -
* To establish a cause and effect relationship, information about
sequence of events is required, i.e. time sequence is required.
* But, cross-sectional study is based on a single examination of a
cross-section of population Exposure and disease are measured at
the same point of time, it is not possible to distinguish whether the
exposure preceded or followed the disease.
* So, information about sequence of events can not be collected --
> cause and effect relationship can not be established. o However,
distribution patterns may suggest causal hypothesis which can be
tested by analytic study -4 Etiological hypothesis can be made.
* So, by cross sectional study etiological hypothesis can be made,
i.e. hypothesis about cause and effect can be made but it can not be
confirmed (established). It is confirmed by analytic studies.
77. Using a new technique, Hb was estimated
in a blood sample. The test was repeated
for 10 times. The reports were : 9.5, 9.2,
9.4, 9.6, 9.7, 9.9, 10.2, 10.3, 10.5, 12.1.
Accurate value of Hb was estimated by
standard tests to be 10.2. The new
technique has ?
a) High validity and high reliability

b) Low validity and low reliability

c) High validity and low reliability

d) Low validity and high reliability

Correct Answer - B
Ans. is 'b' i.e. low validity and low reliability
Validity (Accuracy)
* The validity of a test is the extent to which it accurately measures
which it purports to measure. In other words its the ability to show
which individuals have the disease in question and which do not.
* Validity has two components - sensitivity & specificity. To be truly
valid, a test should be highly sensitive, specific and unbiased.
In this question the test results are not accurate (accurate value is
10.2) Test has low validity. Reliability (or Repeatability,
reproducibility or precision)
Reliability or precision or repeatability is the level of agreement
between repeated measurements of the same variable i.e. a highly
reliable test must give consistent results when repeated more than
once on the same individual or material, under the same conditions.
In this question test is not showing consistent results --> Test has
low reliability.
78. In a study in UK, an association was
found between sale of antiarrythmic drug
and an increase in deaths due to asthma.
This is an example of?
a) Ecological study

b) Cohort study

c) Case reference study

d) Experimental study

Correct Answer - A
Ans. 'a' i.e., Ecological study
Ecological studies (Correlation Studies)
* Ecological study is an epidemiological study in which the unit of
analysis is a population rather than an individual. o
Therefore, ecological study utilizes data at the population level
rather than the individual level.
* The association between a disease and exposure is studied,
e.g., there is a positive correlation between fat
consuption an breast cancer in many nations. Conclusion is that fat
consumption is a risk factor for breast cancer. o Ecological studies
are valuable because they can be done easily and quickly by using
population data that has
already been collecting and seeking correlations between potential
risk factor and various outcomes.
* Ecological studies may be more appropriate than other designs
when studying the impact of an exposure on a
community level.
* An ecological study is normally regarded as inferior to non-
ecological designs such as case-control and Cohort studies because
ecological designs such as case-control and Cohort studies because
it is susceptible to ecological fallacy.
* Ecological fallacy occurs when conclusion are drawn about
individuals from the data that are associated with groups, as
relationship observed for groups may not necessarily hold for
individuals.
In this question association between sale of antiarrhythmic drug and
increase in death due to asthma is given for general population of
`UK' (not for individuals), so it is an example of ecological study.
Other options, case control (case-reference) study, Cohort study and
experimental study (i.e., randomized controlled trials) have already
been explained.
79. Socialization of medicine leads to all
except ?
a) Ensures complete utilization of services by all people

b) Free medical care supported by state

c) Eliminates the competition among physicians in search of


clients

d) Ensures social equity, universal coverage of health services

Correct Answer - A
Ans. is 'a' i.e. Ensures complete utilization of services by all people
Socialized medicine
Socialized medicine describes a system for provision of medical
service and professional education by the state as in state medicine,
but the programme is operated and regulated by professional groups
rather than by the government.
Eliminates the competition among physicians in search of clients.
It ensures social equity, that is universal coverage by health services
Provides free medical care, supported by state
* However mere socialization is not sufficient to ensure utilization of
health services. What is required is 'community participation' which
as envisaged by WHO & UNICEF is "the process by which
individuals and families assume responsibility for their own health
and welfare and for those of the community, and develop the
capacity to contribute to their and the community's development".
State medicine
* State medicine implies provision of free medical service to the
people at government expense.
The above given definition of socialized medicine is according
to Park, 19/e, & 18/e, p 9. The following websites give different view.
They write - 'The term Socialized Medicine is used to describe a
system ofpublicly administered national health care. This system can
range from programs in which the government runs hospitals and
health organizations to programs in which there is national universal
health care.
80. Best method to compare new test and
gold standard test -
a) Correlation study

b) Regressive study

c) Bland and altman analysis

d) Kolmogorov smimov test

Correct Answer - C
Ans. is 'c' i.e., Bland and altman analysis
Bland and Altman analysis
* A bland-altman plot is a method of data plotting used in analysing
the aggrement between two different assay.
* It is identical to a Tukey mean difference plot, which is what it is
still known as in other field but was popularisd in medical statistics
by bland and altman.
* Bland and altman make the point that any two methods that are
designed to measure the same parameter (or property) will have a
good correlation when a set of samples are chosen such that the
property to be determined
vary a lot between them. A high correlation for any two methods
designed to measure the same property is thus in itself is just a sign
that one has chosen a wide spread sample. A high correlation does
not automatically imply that there is good agreement between the
two methods.
Construction of bland -Altman plot
* Consider set of n samples for examples,objects of unknown
volume. Both assays for example, different methods of volume
measurement are performed on each sample, resulting in 2n data
points. Each of the n samples is then
represented on the graph by assigning the mean of the two
represented on the graph by assigning the mean of the two
measurements as the abscissa (x-axis) value, and the difference
between the two values as the ordinate (y-axis) value..
* It can also be used to compare a new measurement technique
with a gold standard even so the interest of the Bland-Altman plot is
contested in this particular case because the error pertains to the
sole new measure.
81. Topical administration of I% Pilocarpine
failed to produce pupillary constriction in
a patient who had a large, dilated pupil
What should be the most probable
reason?
a) Adie's tonic pupil

b) Diabetic III nerve palsy

c) Pharmacological blockade

d) Uncal herniation

Correct Answer - C
Ans. c. Pharmacological blockade (Ref-YANNOFF 3/E PG,1052-
1055)
Pilocarpine I% is a sufficient miotic dose for any eye, but a sphincter
with all its cholinergic receptor blockade by atropine or tropicamide
(i.e. eye with pharmacological blockade) does not constrict with
pilocarpine 1%.
82. Which of the following drug is used
curvently for the prophylaxis of non
infectious uveitis in LUMINATE
programme
a) Cyclosporine

b) Voclosporine

c) Methotrexate

d) lnfli xi mib

Correct Answer - B
B i.e. Voclosporine
- LUMINATE (i.e. Lux Uveitis Multicentre Investigations of a New
Approch Treatment) programme is a scientific drug trial (started in
2007 and completed in 2009) for uveitis. In this programme a pivotal
study LUVENIQ (LX-211) assessed the safety and efficacy of oral
form of next generation calcineurin inhibitor, Voclosporin for
treatment of sight threatening non infectious uveitis (as a steroid
sparing therapy).
- LX - 212 Programme : In this a bioerodible polymer implant
containing voclosporin is implanted, using minimally invasive
approach; which release active ingradient slowly & steadily to the
surface of eye.
- LX - 214 Programme : In this topical formulation (eye drop) of
nanomicelles of voclosporin (which in not normally soluble in
aqueous media) is used for dry eye, atopic kerato conjunctivitis &
blepharitis.
LUMITECT Programme and LUCIDA/ LX-201 (i.e. Lux Corneal
Transplant Implant Development & Advancement of Therapy) Study
: In this silicon matrix ocular (episcleral)implant that steadily releases
therapeutic dose of cyclosporine A is used for prevention of rejection
of corneal transplant.
83. MIZUO phenomenon is seen in
a) Choroideremia

b) Oguchi's disease

c) Fundus flavimaculatus

d) Fundus albipathicus

Correct Answer - B
B i.e. Oguchi's disease
Mizuo phenomenon is seen in Oguchi's diseaseQ. In oguchis
disease the characteristic golden yellowish metallic sheen (color) of
posterior pole (fundus) in light adapted state reverts to normal after
prolonged dark adaptation, a phenomenon described by and named
after Japanese ophthalmologist Mizuo. Re exposure to light results
in the return of golden yellow metallic sheen.
Female carriers of X-linked (XL) retinitis pigmentosa may have
normal fundi or show a golden metallic (tapetal) reflex at macula and
/or small peripheral patches of bone spicule pigmentation. The
inverse Mizuo phenomenon seen in XL-RP is characterized by
appearance of golden sheen in dark (but no golden sheen in light).
84. A young adult presents 2 days after
trauma to the eye with proptosis and pain
in the right eye. On examination, hi is
found to have a bruise on the right eye
and forehead. The most likely diagnosis
is:
a) Fracture sphenoid bone

b) Cavernous sinus thrombosis

c) Internal carotid artery aneurysm

d) Carotico-cavernous fistula

Correct Answer - D
Ans. d. Carotico-cavernous fistula (Ref KANSKI7/E PG94; PARSON
20/E P455,462)
The most likely diagnosis in a young adult with history of trauma to
the eye with proptosis and pain in the right eye and bruise on the
right eye and forehead is Carotico-cavernous fistula
85. Best test to determine etiology of SAH
a) Enhanced CT

b) Unenhanced CT

c) Intra arterial digital Substraction Angiography

d) MRI

Correct Answer - C
C i.e. Intra arterial digital Substraction
Once dx is done by CT Scan; digital substraction angiography (DSA)
is done to determine etiology.
DSA is most sensitive & best inv. for determining etiologyQ.
Now DSA is being replaced by noninvasive methods as MRA (MRI -
angiography) & CTA (CT - Angiography).
86. Which is not a feature of occulomotor
nerve palsy?
a) Miosis

b) Difficulty in accommodation

c) Superior gaze palsy

d) Diplopia

Correct Answer - A
A i.e., Miosis
87. Percentage of endothelial cell loss during
Descent's membrane stripping in
automated penetrating keratoplasty
a) 0-5%

b) 10-15%

c) 30-40%

d) 50-60%

Correct Answer - C
C i.e. 30-40%
The incidence of endothelial cell loss in donor tissue of Descemet
stripping endothelial keratolplasty is nearly 36%, mainly in the
central area of graft. Compared with penetrating keratoplasty, EK
involves more donor-tissue manipulation including lamellar
dissection and folding, which could potentially damage donor
endothelium. On the other hand EK grafts are usually larger (8-9 mm
diameter compared with typical PK diameter of 7 - 8mm) and
provide a larger reservoir of healthy donor endothelium. However,
several recent reports suggest that endothelial cell loss in first few
years after EK is similar to that experienced after PK.
88. A 25-year-old lady presents with sudden
severe bilateral loss of vision, more so on
the right side, with no perception of light.
Rest of the examination including
pupillary reflexes, fundus and optokinetic
nystagmus are normal. She was able to
touch the tips of her fingers with her right
eye closed but not with her left eye
closed. The most like1 tgnosis is:
a) Optic neuritis

b) Anterior ischemic optic neuropathy

c) CMV retinitis

d) Functional vision loss

Correct Answer - D
Ans. d. Functional vision loss (Ref Kanski 6/e p 477, 790, 792-793)
The most likely diagnosis in this young lady who presents with
sudden severe bilateral loss of vision, more so on the right side, with
no perception of light and normal examination findings including
pupillary reflexes, fundus and optokinetic nystagmus, who is able to
touch tips of her finger with right eye closed but not with left eye
closed, is functional vision loss.
89. %lost radio-resistant cells in retina
a) Retinal pigment epithelium

b) Ganglion cell layer

c) Rods and cones

d) Bipolar cells

Correct Answer - B
Ans. b. Ganglion cell layer (Ref Radiation Retinopatliv. ,(1991) 5.
239-251)
Ganglion cells are an example of highly specialized cells that have
undergone extensive maturation, are the most radioresistant cells in
the retina.
90. All of the following occur in herpes
zoster ophthalmicus except
a) Anterior stromal keratitis

b) Pseudodendritic keratitis

c) Sclerokeratitis

d) Endothelitis

Correct Answer - C
C i.e. Sclerokeratitis
Herpes Zoster ophthalmicus commonly causes acute epithelial
keratitis [i.e. punctuate epithelial keratitis (in 50%) & pseudo
dendritic keratitis (50%)], nummular (anterior stromal) keratitis (40%)
disciform (deep stromal) keratitis or endothelitis (34%) and
serpiginous ulceration (7%). However, sclera-keratitis (limbal
vascular keratitis) is least common (1%)Q.
Herpes Zoster Ophthalmicus (HZO) Corneal Disease (Keratitis)
It can result in significant vision loss and manifests in 5 basic clinical
forms
1. Epithelial Keratitis
It may be acute (develop in >50% cases) or chronic (-8% patients).
Punctate epithelial keratitis is earliest finding and presents with
multiple, fine, swollen, raised intraepithelial lesions located
paracentrally or at the limbus which stain intensely with rose bengal
but only mildly with fluorescein. These contain live virus and may
resolve into elevated dendriform epithelial lesion referred to as
pseudo dendrites. (Pseudodendritic keratitis)
Pseudodendrites are typically smaller than dendrites, and lack
terminal end bulb formations. They are transient and usually resolve
within 2 weeks after the cutaneous eruption, and steroids have little
to no effect.
to no effect.
Chronic epithelial keratitis is characterized by coarse pleomorphic
epithelial mucous plaques appearing 1 weeks to 1 year (usually 3-4
months) after the skin lesions. They are a/w diffuse anterior stromal
haze and debridement of plaque is not a/w any damage to
underlying epithelium.
2. Nummular (Anterior Stromal) Keratitis
Is earliest finding of corneal stromal involvement & present during,
2^d week of disease in 25-40% of patients.
- It is characterized by multiple, fine, granular, (usually) transitory
infiltrates in anterior stroma in areas previously affected by either
punctuate or pseudodendrites.
3. Dissiform (Deep stromal) Keratitis or Endothelitis
Develop 3-4 months after initial acute phase & is usually preceeded
by acute epithelial or anterior stromal keratitis.
Central, well defined, disc shaped area of diffuse deep stromal
edema without vascularization, corneal edema & anterior chamber
inflammation may be prominent features
4. Limbal Vascular Keratitis (Sclero-Keratitis)
5. Neurotrophic Keratitis
91. Ozurdex is?
a) 0.5 mg triamcinolone

b) 0.7 mg dexamethasone

c) 0.6 mg betamethasone

d) 0.7 mg Triamcinolone

Correct Answer - B
Ans is b' i.e. 0.7 mg dexamethsone
Ozurdex
* Ozurdex is an intravitreal implant containing 0.7 mg (700µg)
dexamethasone in the solid polymer drug delivery system (NOVAD
UR).
* Ozurdex is specifically indicated in macular edema following
branch retinal vein occlusion (BRVO) or central retinal vein occlusion
(CRVO).
* Recently, efficacy of ozurdex in the treatment of intermediate and
posterior uveitis has also been demostrated.
Mechanism of action
* Dexamethasone is a potent corticosteroid and suppress
inflammation by inhibiting multiple inhibitory cytokines resulting in
decreased edema, fibrin deposition, capillary leakage and migration
of inflammatory cells.
Adverse effects and contraindications
Intraocular pressure increase, conjunctival hemorrhage, eye pain,
conjunctival hyperemia, ocular hypertension, cataract, vitreous
detachment, headache.
* Contraindicated in infections, advanced glaucoma, hypersensitivity.
Other important drug delivery system for eye
Medidur: Delivers Fluocinolone acetonide Retisert: Delivers
Fluocinolone acetonide Vitrasert: Delivers ganciclovir
Fluocinolone acetonide Vitrasert: Delivers ganciclovir
92. Which of the following is not true?
a) William syndrome consists of precocious puberty, mental
retardation and obesity

b) In absence of sunlight daily requirement of vitamin D is 400-600


IU

c) 1 alpha hydroxylation occurs in kidney

d) 25 alpha hydroxylation occurs in liver

Correct Answer - A
William syndrome consists of precocious puberty, mental
retardation and obesity [Ref: Nelson 17/e 215;
http://www.jacn.org/egi/content/full/18/2/122
There is confusion between first 2 options. Last 2 options are well
understood. (See the below given flow chart) Let's see first option
first i.e. William syndrome consists of precocious puberty, mental
retardation and obesity
William syndrome is a rare genetic condition caused due to
mierodeletion of a portion of chromosome 7(7(111. 23) that
involves the elastin gene.
The clinical manifestations include a - Distinct facial appearance, -
Cardiovascular anomalies, - Hypercalcemia, and
- Characteristic neurodevelopmental and behavioral profile.
Virtually all cases have typical facial features that can be recognized
even at birth.
All children and adults with Williams's syndrome have some
combination of the following facial features: short upturned nose, flat
nasal bridge, long philtrum, flat malar area, wide mouth, full lips,
dental malocclusion and widely spaced teeth, micrognathia, stellate
irides, and periorbital fullness. The voice may be harsh. Nails tend
to be hypoplastic and the skin soft and lax, and the hallices have a
valgus deviation. "Elfin fades" is considered a pejorative term, and
its use should be discouraged.
Short stature
Cardiovascular abnormalities e.g. supraventricular aortic stenosis
Children with Williams's syndrome are described as overly friendly,
hyperactive, inattentive, and hypersensitive to loud sounds or certain
types of sounds. Older children and adults have an outgoing
"cocktail party" personality. Lack shyness with strangers.
Relatively good verbal skills.
Mild-to-moderate mental retardation,
Very deficient visuo-spatial abilities
Approximately 70% also suffer from attention deficit disorder.
Though precocious puberty and obesity may also be seen in
William syndrome, they are not the key features of this syndrome.
Hence I would choose option a.
According to Park 19/e Daily requirement of vitamin D is:
Adults 2.5 mcg or 100 IU
Infants and children 5.0 mcg or 200 IU
Pregnancy and lactation 10 .0 mcg or 400 IU
Second option i.e. In absence of sunlight daily requirement of
vitamin D is 400-600 I U
According to the article "Perinatal Vitamin D and Calcium Status of
Northern Canadian Mothers and Their Newborn Infants" published in
Journal of the American College of Nutrition, Vol. 18, No. 2, 122-126
(1998)
Daily requirement of vit D, has been a subject of controversy. For
adults exposed to adequate sunlight, the latest recommended intake
is 2.5 pg(100 1U) in Canada and 5.0 pg(200 1U) in the US. During
pregnancy the daily recommended intake of vitamin D remains to be
2.5 pg in Canada [2], whereas in the US, it is increased to 10 [tg(400
IU). Since a primary source of the vitamin is believed to be the
sunlight, its dietary recommendations vary for populations not
exposed to sufficient sunlight. Some recommend 15 pg/day (600 10
while others have estimated that 12.5 pg/day(500 IU) is sufficient in
the absence of sunlight.
Although there are articles on net which quote different values, I
would still go with option a as the answer. For option c and d:
Synthesis and metabolism of Vitamin D
Calcidiol may undergo 24-hydroxylation to yield an inactive
metabolite, 24, 25-dihydroxy vit. D mediated by vit. D 24 -
hydroxylase.
1, 25 (OH)2D is the major inducer of vit-D-24 hydroxylase, thus
promotes its own inactivation, thereby limiting its biologic effects.
93. A 30-year-old gentleman has exce' cep at
work attributed to sleep discomfort at
night. He also has recent history of falling
while partying with friends. What are the
other features that can be seenassociated
with his condition?
a) Paralysis during sleep-wake transition with hallucinations

b) Snoring with witnessed sleep apnea

c) Pain in the legs before going to sleep

d) Generalized seizures in the wake state

Correct Answer - A
Answer: a. Paralysis during sleep-wake transition with hallucinations
(Ref. Harrivon 19/c p189, 18/e p265
History of fall is highly suggestive of atonia/paralysis and may be
due to Narcolepsy.
94. All of the following are Preleukemic
conditions, Except:
a) Paroxysmal Nocturnal Haemoglobinuria (PNH)

b) Paroxysmal Cold Haemoglobinuria (PCH)

c) Myelodysplasia

d) Aplastic anemia

Correct Answer - B
Answer is B (Paroxysmal Cold Haemoglobinuria (PCH))
Paroxysmal Cold Hemoglobinuria (PCH) is a benign self-limiting
autoimmune hemolytic anemia. PCH does not undergo leukemic
transformation and is hence not considered a Preleukemic condition.
Recovery is the rule in Paroxysmal Cold Hemoglobinuria.
Paroxysmal Nocturnal Haemoglobinuria may be considered a
Preleukemic condition
PNH may evolve into Aplastic anemia and PNH may manifest itself
in patients who previously have Aplastic Anemia. Rarely (estimated
1-2% (Ian cases) PNH may terminate in Acute Myeloid Leukemia'. –
Harrison 18th/883
Myelodysplasia (Myelodysplastic Syndrome) is a Preleukemic
condition
Myelodysplastia is a clonal hematopoietic stem cell disorder leading
to impaired cell proliferation and differentiation Cvtogenetic
abnormalities are found in approximately one half of patients and
some of the same specific lesions are also seen in frank leukemia.
The type and number of cytogenetic abnormalities strongly correlate
with the probability of leukemic transformation and survival'
Aplastic Anemia may be considered a Preleukemic condition
`Recent studies have shown that long term survivors of acquired
aplastic anemia may be at high risk of subsequent malignant
diseases or late clonal hematological diseases often years after
successful immunosuppressive therapy. One Hypothesis has
postulated that aplastic anemia is a Preleukemic condition'.
95. 72-y ear-olo gentleman with normal renal
functions presents with new onset focal
seizures. Which of the following is the
best drug to manage the patient?
a) Sodium valproate

b) Oxcarbazepine

c) Leviteracetam

d) Pregabalin

Correct Answer - B
Answer: b. Oxcarbazepine (Ref: Harrison 194, p2552. 18/e pc
1)3262)
Among the options provided, oxcarbazepine is the best drug to
manage this old patient with normal renal functions presents with
new onset focal seizures
96. False regarding Alzheimer's disease (AD)
is:
a) Number of senile neural plaques correlates (increases) with age

b) Presence of tau protein suggest neurodegeneration

c) Number of neurofibrillary tangles is associated with the severity


of dementia

d) Extracellular inclusions (lesions) can occur in the absence of


intracellular inclusions to make

Correct Answer - D
D i.e. Extracellular inclusions (lesions) can occur in the absence of
intracellular inclusions to make pathological diagnosis of AD
Characteristic cognitive impairments in Alzheimer's disease include
amnesia, aphasia, agnosia, and apraxia (the 4 As)Q. These four
impairments are also included in DSM-IV TR diagnostic criteria of
Alzheimer's disease. However, there appears no cognitive functions
that are truely preserved in AD. Visuo-spatial difficulties commonly
occur in the middle stages of the disorder and may result in
topographical disorientation, wandering and becoming lost.
Difficulties with calculation (acalculia), attention and cognitive
planning all occur.
- For neuropathological diagnosis of AD, h/o dementia, extracellular
neuritic senile (amyloid) plaques and intracellular NFTs all are
required. Number of senile plaque increases with age, number of
NFTs correlates with severity of dementia and presence of tau (r)
protein suggest neurodegenerationQ.
It is the most common cause of dementiaQ. It causes gradually
progressive cortical (parieto-temporal) dementiaQ beginning with
memory impairment and spreading to language and visuospatial
deficits over a prolonged courseQ.
97. Which of the following is not a Limb
Girdle Dystrophy:
a) Sarcoglycan dystrophy

b) Dystrophin dystrophy

c) Dysferlin dystrophy

d) Calpain dystrophy

Correct Answer - B
Answer is B (Dystrophin dystrophy):
Calpain - J, Sarcoglycans (y, a, fl and 0 and dysferlin are all
examples of defective gene / locus in Limb girdle dystrophies.
Dystrophin is associated with Duchenne and Becker's type of
muscular dystrophics and is not associated with Limb girdle
dystrophies.
The Limb girdle muscular dystrophies (LGMD) typically manifest with
progressive weakness of pelvic and shoulder girdle musculature.
Respiratory insufficiency from weakness of diaphragm and
cardiomyopathy may occur. However unlike Duchenne dystrophies
(Dystrophin) intellectual function remains intact. - Harrison
98. In Alzheimer's disease (AD) which of the
following is not seen:
a) Aphasia

b) Acalculia

c) Agnosia

d) Apraxia

Correct Answer - C
Ans. None > C. Agnosia tRej. Kaplan anal Jaadock s Synopsis of
Psychiatry 10/c pi-t?
Aphasia, Apraxia, Acalculia and Agnosia may all be seen in
Alzheimer's Dementia. Agnosia in Alzheimer's disease usually
presents late in the disease and is not included in the ICD-10
Diagnostic criteria for dementia in Alzheimer's disease with early
onset and hence may be selected as the single best answer by
exclusion
99. A 29-year-old, 4 months pregnant
primigravida has history of juvenile
myoclonic epilepsy. She has been
regularly taking sodium valproate and
now presents requesting for an opinion
for a change in her anti-epileptic
treatment. What would you sugeets her?
a) Immediately taper off valproate and start lamotrigine

b) Switch to carbamazepine

c) Continue valproate and serial drug monitoring in blood

d) Add lamotrigine to valproate

Correct Answer - C
Answer: c. Continue valproate and serial drug monitoring in blood
(Ref Harrison I9/e p2544, 18/e p3269)
Continue valproate with serial drug monitoring in the blood of the 4-
month old patient having history of juvenile myoclonic epilepsy.
100. Which of the following is used in the
treatment of Multiple Sclerosis:
a) Interferon Alpha

b) Interferon Beta

c) Infliximab

d) Interferon gamma

Correct Answer - B
Answer is B (Interferon Beta) :
Interferon Beta should be considered in patients with Relapsing /
Remitting MS (RRMS) and Secondary Progressive MS (SPMS) with
superimposed relapses.
Interferon Beta reduces the attack rate in M.S. patients and also
improves disease severity measures and disease burden.
Interferons
Interferons are cytokines that exhibit antiviral activities,
immunomodulating properties and antiproliferative properties.
DNA recombinant technology has made available highly purified a,
(3 and y interferons.
IFNs are not available for oral administration but must be given
intramuscularly, subcutaneously or intravenously.
Interferon Alpha (IFN Interferon Beta
Interferon (IFN y)
a) HEN(i)
Considered for : Considered for : Considered for :
• Hepatitis B, Multiple • Osteopetrosis , (H /
Hepatitis C Sclerosis 2282)
• Mucocutaneous
(suggested benefit )
manifestations of
Behcet's syndrome (H / 3 Approved
Behcet's syndrome (H / 3 Approved • Systemic sclerosis
2014) Agents ((H/1987)
• Carcinoid syndrome • Aronex (modest improvement but
(H / 2226) (IFN 3la) recurrence)
• Chronic myeloid • Rebif (IFN • Idiopathic pulmonary
leukaemia (CML) (H (3 I a) fibrosis
Betaseron (IFN
/ 640) (therapeutic trial) H/ 1556
(31b)
• Follicular lymphoma Chronic granulomatous
(H / 651) disease leprosy,

• Malignant non tuberculosis
melanoma mycobacteria & visceral
(Adjuvant) (H / 502) leishmaniosis (H/357)
• Essential • Multiple myeloma

thrombocytosis (H / 660)
• Renal cell

carcinoma (H / 542)
• Splenornegaly (H /

628)
Also Note :
Interferon is not curative for any tumor but can induce partial
responses in (Harrison 16th / 482)
Follicular lymphoma Hairy cell leukaemia CML
Melanoma
Kaposi's sarcoma
IFN y has been successfully used in the treatment of Leprosy,
Nontuberculous Mycobacteria and Visceral Leishmaniosis. -
Harrison I6th / 357
101. All of the following are approved for the
treatment of relapsing-remmitting
multiple sclerosis (RRMS) subtype,
except-
a) IFN - beta - la

b) IFN - beta - lb

c) Glatiramer

d) Mycophenolate

Correct Answer - D
Ans. is 'd' i.e. Mycophenololate
Treatment of RRMS
* Treatment of RRMS is divided into:?
(i) In acute attack: Corticosteroids are given
(ii) Prophylaxis of acute attack (relapse) during remission: Disease
modifying agents for MS are used to reduce the biological activity.
Treatment is started by IFN-131a or IFN-131b or Glatiramer or
fingalimod. If there is poor response or intolerance to these drugs,
Natalizumab is started.
Treatment of progressive subtype of MS
* Treatment is started with l FN-fil a or IFN-$lb.
* If there is poor response or intolerance, one of the following
should be considered: (i) Mitoxantrone; (ii) Azathio? prine; (iii)
Methotrexate (iv) IV immunoglobulin; (v) Pulse cyclophosphamide;
(vi) Pulse methylprednisolone.
102. Which of the following is the only drug is
found to be beneficial in halting the
progression on EDSS of multiple
sclerosis:
a) Glatiramer

b) IFN-beta

c) Natalizumab

d) Methotrexate

Correct Answer - C
Ans is 'c' i.e. Natalizumab
* EDSS (Expanded disability status scale) is a method of
quantifying disability in multiple sclerosis.
* EDSS steps 1.0 to 4.5 refer to people with MS who are fully
ambulatory. EDSS steps 5.0 to 9.5 are defined by the impairment in
amubulation. EDSS 10 is defined as death due to MS.
* Natalizumab silences disease activity and rapidly improves
disability status and walking performance, possibly through delayed
relapse recovery in patients with RRMS who had shown a high level
of disease activity under other disease modifying drugs for multiple
sclerosis.
103. Which of the following clinical features
of demyelinating myelopathy least likely
suggests a progression to multiple
sclerosis?
a) Complete cord transection

b) Bilateral visual loss

c) Absence of oligoclonal bands

d) Poor prognosis

Correct Answer - C
Answer: c. Absence of oligoclonal bands (Ref Harrison I9/e p2656.
18/e p3372)
Oligoclonal bands are variable; but when bands are present, a
diagnosis of MS is more likely, thus absence of oligoclonal bands
suggests a less likely progression to MS.
104. All of the following are neurologic channelopathies except:
a) Episodic ataxia type 1

b) Spinocerebellar ataxia 1

c) Familial hemiplegic migraine

d) Hypokalemic periodic paralysis

Correct Answer - B
Spinocerebellar ataxia (SCA) type 1 is not a neurological channelopathy. But SCA type 6
is a neurological channelopathy. Channelopathies refers to a wide variety of neurologic
diseases caused by dysfunction of ion channels in the excitable membranes. These are
due to inherited mutations of genes encoding ion channels in muscle, neurons and glia.
Severe myoclonic epilepsy of infancy, episodic ataxia type 1, 2, familial hemiplegic
migraine, spinocerebellar ataxia type 6 are examples of neurologic channelopathy.

Ref: Harrison’s Internal Medicine, 18th Edition, Chapter 373; A-Z of Neurological Practice:
A Guide to Clinical Neurology By Andrew J. Larner, 2nd Edition, Page 122
105. The following are components of Brown
Sequard syndrome except :
a) Ipsilateral extensor plantar response

b) Ipsilateral pyramidal tract involvement

c) Contralateral spinothalamic tract involvement

d) Contralateral posterior column involvement

Correct Answer - D
Answer is D (Contralateral posterior column involvement):
Brown – Sequard syndrome or hemisection of the spinal cord leads
to loss of joint position and vibratory sense (posterior coluntn
movement) on the ipsilateral side and not on the contralateral side. –
Harrison 16th / 2441, 144
Brown sequard syndrome: Hemisection of spinal cord
Ipsilateral involvement of corticospinal tract : ipsilateral loss of motor
power.
Ipsilateral involvement of posterior column : ipsilateral loss of joint
position and vibratory sense.
Contralateral involvement of spinothalamic tract : contralateral loss
of joint position and vibratory sense.
Segmental signs such as radicular pain muscle atrophy or loss of
deep tendon reflexes arc unilateral (Lower motor neuron signs at
level of lesion).
106. Hyperaldosteronism is associated with
all except
a) Hypermatrenia

b) Hypokalemia

c) Hypertension

d) Metabolic acidosis

Correct Answer - D
D i.e. Metabolic acidosis
Hyperaldosteronism is assosciated with metabolic alkalosis along
with hypernatremia, hyponatremia,hypokalemia & hypertension.
Also know
Primary Aldosteronism also k/a Conn's syndrome, is most commonly
caused by unilateral adrenal adenoma
It is characterised by
- Diastolic hypertension without oedema
- Low plasma renin activity levels
Secondary aldosteronism with high renin activity is seen in cirrhosis,
CHF, nephrosis & primary reninism (renin producing tumors)
Aldosterone primarily acts on the principal cells (P cells) of the
collecting ducts.
107. All of the following statements about
SIADH are true except:
a) Serum sodium is low, typically < 135 meq/1

b) Urinary sodium excretion is low / normal

c) Water loading test may be used

d) Vaptans are new FDA approved agents for treatment of SIADH

Correct Answer - B
Answer is B (Urinary sodium excretion is low / normal):
SIADH is typically characterized by high urinary sodium (Increased
rate of excretion of. odium).
Syndrome of Inappropriate ADH Secretion (SIADH) is associated
with increased secretion of vasopressin (ADH), which leads to
increased absorption of water producing dilutional hyponatremia
(serum sodium typically < 135 meq/l) along with concentrated or
hyperosmolar urine. Excessive retention of water stimulates
compensatory mechanisms that enhance Watriuresis'.
Natriuresis results in increased urinary sodium excretion rate
increased urinary sodium concentration) and is believed to
compensate for increased volume from inappropriaye ADH secretion
preventing a state of clinical hypervolemia, hypertension or edema.
SIADH is associated low serum sodium levels (
Cardinal features of SIADH include:
Hyponatremia (dilutional hyponatremia with Na+ < 135 mmo1/1)
Decreased plasma osmolality (increased urine osmolality > 150 m
osm).
High Urine sodium (over 20 meq4)
Low Blood urea Nitrogen < 10 mg/L
Hypouricemia (
Clinical Euvolemia
- Absence of signs of hypervolemia (edema, ascitis)
- Absence of signs of hypovolemia (orthostatic hypotension,
tachycardia, features of dehydration)
Absence of cardiac, liver or renal disease
Normal thyroid and adrenal function
A high BUN suggests a volume contracted state and excludes a
diagnosis of SIADH.
Water loading test may be used to help diagnosis of SIADH
Water loading test is recognized as a 'supplemental criteria' in
establishing a diagnosis of SIADH.
The 'Water Loading Test' is of value when there is uncertainity
regarding the etiology of modest degrees hypo-osmolality in
euvolemic patients but it does not add useful information if the
plasma osmolality is < 275 mOsm/kg H2O
Abnormal Water Loading
Test Objective / Principle
Test Criteria
Fixed quantity of water is given to a
Abnormal water load
patient and the amount of
urine produced and changes in Inability to excrete at least
blood/urine osmolality are 90% of a 20m1/kg
recorded. water load in 4 hours, and/or
Inability to excrete a normal water Failure to dilute urine (osm)
load (decreased urinary to < 100 mOsm/kg
output) or failure to dilute urine is
H-,O.
considered abnormal
Vaptans are new FDA approved agents for treatment of SIADH
`Vaptans' are a new class of drugs that have emerged for treatment
of hyponatremia. These medications act as Vasopressin receptors
antagonists blocking the action of AVP in renal tubule, pituitary or
smooth muscles depending upon receptor selectivity.
Conivaptan , itravenous use)
Conivaptan is a combined V1/V2 receptor antagonist
is FDA approved for short-term intravenous use fin-
treatment of hospitalized patients with SIADH.
Tolivaptan (Oval use)
Tolivaptan is a V2 receptor antagonist that has
received FDA approval far oral use
100 mOsmol/kg H20 with normal renal function) at some level of
hypoosmolality. (Urinary concentration must be inappropriate for
Plasma Hyposmolality) • Clinical euvolemia, as defined by the
absence of signs of hypovolemia (orthostasis, tachycardia,
decreased skin rurgor, dry mucous membranes) or hypervolemia
(subcutaneous edema, ascites). • Elevated urinary sodium excretion
while on normal salt and water intake. • Absence of other potential
causes of euvolemic hypoosmolality: hypothyroidism,
hypocortisolism (Addison's disease or pituitary adrenocorticotropic
hormone [ACTH] insufficiency) and diuretic use. Supplemental •
Abnormal water load test (inability to excrete at least 90% of a 20
mL/kg water load in 4 hours and/or failure to dilute Uosm to < 100
mOsmol/kg H-0). • Plasma AVP level inappropriately elevated
relative to plasma osmolality. • No significant correction of serum
[Nal with volume expansion but improvement after fluid restriction "
align="left" height="259" width="655">Syndrome of Inappropriate
ADH Secretion (SIADH): Concept Review
Differential Diagnosis of Hyponatremia Based on Clinical
Criteria for the diagnosis of syndrome of innapropriate
antidiuretic hormone` secretion
Essential
Decreased effective osmolality of the extracellular fluid (13„,,, < 275
mOsmol/kg H20).
(True hypoosmolality must be present and hyponatremia secondary
to pseudohyponatremia or hyperglycemia must be excluded)
Inappropriate urinary concentration (Uosm > 100 mOsmol/kg H20
with normal renal function) at some level of
hypoosmolality. (Urinary concentration must be inappropriate for
Plasma Hyposmolality)
Clinical euvolemia, as defined by the absence of signs of
hypovolemia (orthostasis, tachycardia, decreased skin rurgor, dry
mucous membranes) or hypervolemia (subcutaneous edema,
ascites).
Elevated urinary sodium excretion while on normal salt and water
intake.
Absence of other potential causes of euvolemic
hypoosmolality: hypothyroidism, hypocortisolism (Addison's disease
or pituitary adrenocorticotropic hormone [ACTH] insufficiency) and
diuretic use.
Supplemental
Abnormal water load test (inability to excrete at least 90% of a 20
mL/kg water load in 4 hours and/or failure to dilute Uosm to < 100
mOsmol/kg H-0).
Plasma AVP level inappropriately elevated relative to plasma
osmolality.
No significant correction of serum [Nal with volume expansion but
improvement after fluid restriction
Assessment of Extracellular Fluid Volume (ECFV)
Clinical Type 1, Type II, Type III, SIADH
Findings Hypervolemic Hypervolemic Hypervolemic Euvolemic
History
CHF,
cirrhosis, or Yes No No No
nephrosis
Salt & water
No Yes No No
loss
ACTH- No No Yes No
cortisol
deficiency
and/or
nausea and
vomatiting
Physical examination
Generalized
edema, Yes No No No
ascites
Postrlial
May be May be May be" No
hypotension
Laboratory
BUN, Low-
High-normal High-normal Low-normal
creatinine normal
Low-
Uric acid High-normal High-normal Low-normal
Low-
Uric acid High-normal High-normal Low-normal
normal
Serum
Low-normal Low-normal Normal Normal
potassium
Serum
Low-normal High-normal Normal Normal
albumin
Serum
Normal-high Normal-high Low Normal
cortisol
Plasma
renin High High Low Low
activity
Urinary
sodium
Low Low High High
(Meg unit of
time)
108. Diabetes control is best monitored by:
a) Serum glucose

b) Post prandial blood glucose

c) HbA1C

d) HbA2C

Correct Answer - C
Answer is C (HbA1c):
Measurement of glycate hemoglobin (HbA1C) is the standard
method for assessing long term glycemic control.
The gold standard indicator of glycemic control in patients
with diabetes mellitus is hemoglobin A1c (HbA1c)
Glycated hemoglobin (HbA1C) should be measured in all individual
with DM as part of their comprehensive diabetic care.
109. Addison's disease was first reported by
Thomas Addison. It is still being widely
reported from various parts of the world.
Which of the following is the most
common cause of Addisons's disease in
India?
a) Tuberculous adrenalitis

b) Post-partum pituitary insufficiency

c) Autoimmune adrenalitis

d) HIV infection of adenals

Correct Answer - A
Answer: . a. Tuberculous adrenalitis (Ref API 7/e 171073; flm-ricon
19/r n2323, 18/e p2955, '141)Mostcommon cause of Addison's
disease in India is tuberculous adrenalitis.
110. With reference to the optimal
management of patients with sepsis in
the intensive care unit, all of the
following interventions are evidence-
based except:
a) Low tidal volume during assisted ventilation prevents acute lung
injury

b) For outcome of goal-oriented management of sepsis patients,


monitoring of central venous pressure, hourly urine output and
blood pressure is adequate

c) Intensive blood glucose monitoring and prevention of


hyperglycemia improves survival in critically ill patients

d) The use of drotrecogin-alpha is restricted to severely ill patients


with APACHE score > 25

Correct Answer - C
Anser: . c. Intensive blood glucose monitoring and prevention of
hyperglycemia improves survival in critically ill patients (Harrison
19/e p1751, 18e/pg 2954-2957)
Low tidal volume during assisted ventilation prevents acute
lung injury:
The results of recent studies favor the use of low tidal volumes 6
mL/kg of ideal body weight as low as 4 mL/kg if plateau pressure
exceeds 30 cm H2O in order to reduce the risk of ventilator induced
Acute Lung Injury (ALI).
111. Which of the following Slatements is not
true?
a) Parathyroid hormone-related protein is responsible for causing
hypercalcemia in cancer patients

b) The unionized fraction of calcium in the plasma is an important


determinant of PTH secretion

c) Mg2+ influences PTH secretion in the same direction as Ca2+


but is a less potent secretagogue

d) Ca2+influences PTH secretion by acting on a calcium sensor G-


protein coupled receptor located in the parathyroid gland

Correct Answer - B
Ans. b. The unionized fraction of calcium in the plasma is an
important determinant of PTH secretion
"Circulating ionized calcium acts directly on the parathyroid glands in
a negative feedback fashion to regulate the secretion of PTH.
"Magnesium is required to maintain normal parathyroid secretory
responses. Impaired PTH release along with diminished target
responses to PTH account fir the hypocalcaemia that occasionally
occurs in magnesium deficiency
112. A 30 years old female, RBC Counts 4.5
million, MCV 55 fl, Tc 8000, no history of
blood transfusion?
a) Iron deficiency anemia

b) Thalassemia major

c) Thalassemia minor

d) Megaloblastic anemia

Correct Answer - C
Ans. is 'c' i.e., Thalassemia minor
The clues in this question are :
(i) Normal RBC Count (iii) Normal WBC
count
(ii) Microcytosis (reduced MCV) (iv) No History of
blood transfusion
Option b & d' can be easily ruled out :
(i) Thalassemia major is transfusion dependent and it is not
possible to survive upto the age of 30 years without blood
transfusion.
(ii) In megaloblastic anemia MCV is increased
Now we are left with Thalassemia minor and iron deficienay anemia,
both of them cause microcytic anemia (MCV). These can be
differentiated by following fact :?
o In thalassemia minor the RBC count is normal because the
marrow can keep on producing the cell at normal rate but it cannot
fill them with hemoglobin. On the other hand in iron deficiency
anemia the RBC production is also impaired and the decrease in
RBC count is in proportion the decrease in hemoglobin.
There are many indices to differentiate between iron deficiency
There are many indices to differentiate between iron deficiency
anemia (IDA) and beta-thalassemia (BT)
Index Formula Value for Iron Value for
deficiency beta?
anemia thalassemia
Mentzer index MCV / RBC count >13 <13
Shine and Lal
MCV2 x MCH x 0.01 >1530 <1530
index
England and MCV - RBC - (5 x
>0 <0
Fraser index Hb)- 5.19
Srivastava index MCH / RBC >3.8 <3-8
Green and king MCV2 x RDW x
>65 <65
index Hb/100
Red Cell
MCV x RDW/RBC >220 <220
Distribution
Width index
113. Treatment of choice for Kawasaki
Disease is:
a) IV Immunoglobulins

b) Steroids

c) Dapsone

d) Methotrexate

Correct Answer - A
Answer is A (IV Immunoglobulins):
The treatment of choice in Kawasaki disease is intravenous
immunglobulins
114. Which of the following is the most
specific test for Rheumatoid Arthritis
a) Anti- ccp antibody

b) Anti IgM antibody

c) Anti IgA antibody

d) Anti IgG antibody

Correct Answer - A
Answer is A (Anti - cep Antibody)
'Anti CCP antibodies are the most specific blood test fir rheumatoid
arthritis (specificity –95%)' - CMDT 09/727
Anti-cyclic citrullinated peptide (anti-CCP) antibody testing is
particularly useful in the diagnosis of rheumatoid arthritis, with high
specificity,
presence early in the disease process, and ability to identify patients
who are likely to have severe disease and irreversible damage.
Anti-CCP antibodies have not been found at a significant frequency
in other diseases to date, and are more specific than rheumatoid
factor for detecting rheumatoid arthritis.
- Oxford Journal of Medicine
115. A patient has normal PT and Platelet
Count. The aPTT is increased and factor
VIII levels are observed to be 60 lukIL
(60%). There is no associated history of
bleeding even after a surgical procedure.
The most likely diagnosis is:
a) Factor IX deficiency

b) Factor VIII inhibitors

c) Lupus anticoagulant

d) Thalassemia

Correct Answer - C
Answer is C (Lupus Anticoagulant)
Isolated prolongation of aPTT with no clinical history of bleeding
suggests the presence of Lupus anticoagulant or deficiency of
contact factors (Factor XII, Prekallikrein or HMWK)
116. A 50-year-old asymptomatic man with
established aortic stenosis undergoes
Exercise Stress testing according to
Bruce Protocol. The stress test was
terminated at 11 minutes due to
development of fatigue and dyspnea.
Regional pressure gradient was
observed to be 60 mm Hg between the
two sides of the aortic valve. What is the
best management.
a) Angiogram

b) Aortic valve replacement

c) Aortic Balooning

d) All

Correct Answer - B
Answer is B (Aortic valve replacement )
The patient in question has asymptomatic aortic stenosis but
develops symptoms on exercise (abnormal/positive exercise test).
Also the presence of mean pressure gradient of 60mm Hg put this
patient into the category of 'Very Severe Aortic Stenosis' or 'Critical
Aortic Stenosis'.
Optimal management of asymptomatic severe Aortic Stenosis
continues to be a source of ongoing clinical controversy. Surgical
Aortic Valve Replacement and Watchfull waiting with frequent
reassessments (observation), both continue to be legitimate though
debatable treatment options.
debatable treatment options.
Since the patient in question has 'Very Severe AS' and 'Abnormal
Stress Testing' he should be considered for surgical intervention in
the form of Aortic Valve Replacement (A VR).
50-year-old patient with Asymptomatic Severe Aortic Stenosis


Classification of Aortic Valve Stenosis Severity
Severity Valve Area Mean Pressure
Maximum Aortic
(cm2) Gradient (mm
Velocity (m/sec))
Hg)
Mild 1.5 – 2.0 2.5 – 3.0 <25
Moderate 1.0 –1.5 3.0-4.0 25-40
Severe 0.6-1.0 >4.0 >40
Critical (Extremely
<0.6 >5.0 >50-60
Severe)
117. All of the following statements about
Universal Definition of Myocardial
Infarction are true, Except:
a) Sudden, unexpected Cardiac death with symptoms of Ischemia.

b) Elevation of cardiac biomarkers with new regional wall motion


abnormality

c) Three times increase in Troponin levels after Percutaneous


Coronary Intervention (PCI)

d) Three times increase in Troponin levels after Coronary Artery


Bypass Grafting (CABG)

Correct Answer - D
Answer is D (Three times increase in Troponin levels after Coronary
Artery Bypass Grafting (CABG)
Myocardial Infarction associated with CABG is defined by an
increase in cardiac biomarkers to more than 5 times normal (> 5
times above the 99th percentile).
Universal Defination of Myocardial infarction
118. A 25 years old lady with a history of fever
for 1 month presents with headache and
ataxia.
Brain imaging shows dilated ventricles
and significant basal exudates. Which of
the following will be the most likely CSF
finding?
a) Lymphocytosis, Low Glucose, High protein

b) Lymphocytosis, Normal Glucose, High protein

c) Lymphocytosis, Low Glucose, Normal protein

d) Neutrophilia, Low glucose, Low Protein

Correct Answer - A
Ans. A (Lymphocytosis, Low Glucose, High protein)
Presence of significant basal exudates, together with dilated
ventricles (hydrocephalus) in a young female with a prolonged
history of fever and headache suggests a diagnosis of Tubercular
Meningitis.
Tubercular Meningitis is characterized by Lymphocytic Pleocytosis,
Low Glucose and High Protein within the CSF.
The pathological hallmark of Tubercular Meningitis is the
predominant involvement of basal cisterns that are observed by the
presence of basal inflammatory tissue exudate.
119. A 25 year old lady on treatment for
rheumatoid arthritis has following lab
findings: Hb-9gm/dl, MCV- 55fl, serum
iron-30microgm/d1, feritin​200ng/ml,
TIBC- 298 microgm/dl. What is the most
probable diagnosis?
a) Thalassemia minor

b) Thalassemia major

c) Anemia of chronic disease

d) Iron deficiency anemia

Correct Answer - C
Ans. is 'c' i.e., Anemia of chronic disease
o Information in this question are :-
i) Anemia : Hb- 9 gm/di (normal in female is > 12gm/d1).
ii) Microcytosis : MCV 55 fl (normal is 82-96 fl)
iii) Marginally low serum iron : 30 microgm/dl ( normal value for
female is 26- 170 microgm/dl).
iv) Increased serum ferritin : 200 ng/ml ( normal value for female
is 12-160 ng/ml or microg/L).
v) Normal TIBC : 298 microgram/di ( normal value is 262-474
microgm/dl).
iv) Patient has chronc immune disorder, i.e., RA.
o So, the diagnosis is anemia of chronic disease which can occur in
RA.
Sideroblastic Iron
Iron
anemia refractory
Chronic Thalassemia
iron
iron
Deficiency
deficiency
Disease Trait (a or f3)
anemia
Decreased
MCV/MCH Decreased Decreased Decreased
or
Normal
Serum iron DecreasedDecreasedNormal Increased Decreased
Decreased
TIBC Increased Normal Normal Decreased
or
Normal
Transferrin DecreasedDecreasedNormal Increased Normal
saturation
Serum
DecreasedNormal or Normal Increased Increased
ferritin
Increased
Serum IncreasedNormal Normal Normal
Normal or
transferrin
Increased
receptor
Serum
DecreasedIncreased Normal Decreased Increased
hepcidin
Bone
DecreasedNormal or Normal Normal or Normal or
marrow iron
stores Increased Increased Increased
Erythroblast DecreasedDecreasedNormal Ring forms Increased
iron
120. Which of the following is not. true
regarding mye​lopathy?
a) Sensory loss of facial area

b) Brisk jaw jerk

c) Brisk pectoral jerk

d) Urgency and incontinence of micturition

Correct Answer - B
Ans. b. Brisk jaw jerk
(Ref: De Jongs Neurological examination/ p194, 201, 474.'
Jaw jerk is exaggerated in supranuclear lesions that are above the
mid pons.
121. A young male patient presents with LDL
600mg/dl, triglycerides 140 mg/dl. What
would be the most likely finding on
physical examination
a) Tendon xanthoma

b) Planar xanthoma

c) Lipemia retinalis

d) Tuberoeruptive xanthoma

Correct Answer - A
A i.e. Tendon Xanthoma
Tendon xanthomas are associated with type II hyperlipidemia,
chronic biliary tract obstruction, and primary biliary cirrhosis. Palmar
xanthomata and tuberoeruptive xanthomata (over knees and
elbows) occur in type III hyperlipidemia.
The lipid profile and the lipoprotein electrophoresis reveales
elevated levels of low-density lipoprotein (LDL) cholesterol, and total
cholesterol.
Biopsy from the nodule reveales infiltration of the dermis with
admixture of foam cells, histiocytes and lymphocytes, The lipid
profile of the parents shows a rise in the LDL cholesterol
122. A lady presents with complaints of
abdominla pain. CECT shows bilateral
papillary necrosis. Which of the
following test shall not be done to
investigate the cause of her
papillary necrosis?
a) Urine acidification test

b) Sickling test

c) Urine RT-PCR for TB

d) Bacterial culture of urine

Correct Answer - A
Ans. a. Urine acidification test
(Ref: Harrison 19/e p1860, 18/e p2372)
Urine acidification test is mainly done to diagnose RTA (Renal
tubular acidosis), not the causes of papillary necrosis.
123. Which of the following statement is not
true?
a) Late enhancement on Gadolinium: MRI suggests scar

b) Akinetic area does not benefit from reperfusion

c) Low dose dobutamine may be used to detect hibernating


myocardium

d) Rest reinjection thallium is used to detect hibernating


myocardium

Correct Answer - B
Ans. b. Akinetic area does not benefit from reperfusion
"Hibernating myocardium is a reversible state of contractile
dysfunction observed in the course of chronic ischemia associated
with reduction of coronary blood flow without major evidence of
irreversible myocardial damage (viable myocardium).
The contractile function of hibernating myocardium can be
completely restored after reperfusion.
The contractile function of the hibernating myocardium can be
almost completely restored after reperfusion (revascularization)."-
'Panvascular Medicine' by Topal (Springer) 2002/735.
124. Urine analysis of a patient with
hematuria and hypercalciuria is most
likely to reveal;
a) Isomorphic RBCs

b) RBC casts

c) Nephrotic range proteinuria

d) Eosinophiluria

Correct Answer - A
Ans. a. Isomorphic RBCs
Urine analysis of a patient with hematuria and hypercalciuria is most
likely to reveal isomorphic RBCs.
"Hypercalciuria is a cause of non-glomerular hematuria. RBCs from
a non-glomerular source more closely resemble peripheral blood on
microscopy, with isomorphic RBCs and absence of casts."
125. Peripheral eosinophila with renal failure
is least likely to be possible in:
a) Drug-induced interstitial nephritis

b) Drug-induced interstitial nephritis

c) Atheroembolic renal failure

d) Polyangitis nodosa

Correct Answer - D
Answer. d. Polyangitis nodosa
(ref- Wallach interpretation of diagnostic test 8/e pg 95-96)
Peripheral eosinophilia with renal failure is least likely to be possible
in polyangitis nodosa.
126. A child presents with raised ICT, on CT
scan a lesion is seen around foramen of
Monroe and multiple periventricular
calcific foci. What is the most probable
diagnosis?
a) Central Neurocytoma

b) Ependymoma

c) Subependymal Giant cell Astrocytoma

d) Ganglioglioma

Correct Answer - C
Ans. is 'c' i.e. Subependymal Giant cell Astrocytoma
Both Central Neurocytoma and Subependymal Giant cell
Astrocytoma lesions can be found at the Foramen of Monroe. Both
can cause obstruction at the foramaen of Monroe leading to
ventricular enlargement and raised ICT. But it is the presence of
multiple periventricular calcific foci (calcified subependymal nodules)
which suggest a diagnosis of Tuberous sclerosis with
Subependymal Giant cell Astrocytoma.
Subependymal Giant cell Astrocytoma appears as an enhancing
lesion at the Foramen of Monroe in a patient with Tuberous
Sclerosis.
• Other findings in Tuberous Sclerosis:
Calcified subependymal nodules (98%)
Cortical/subcortical tubers, (70-95%)
White matter lesions along lines of neuronal migration (White
matter radial migration lines representing)
127. Postoperative ileus is most commonly
seen in:
a) Colon

b) Ileum

c) Ileum

d) Stomach

Correct Answer - A
Ans. a. Colon
most abdominal operations or injuries, the motility of the GI tract is
transiently impaired.
Return of normal motility: Small intestine/ (within 24 hours)
>Gastric (48 hours)°> Colonic (3-5 days)°.
Post-operative ileus is most pronouncedin colon.
128. Injury to which of the following vessels
is least likely to result in significant
damage:
a) Renal artery

b) Superior mesenteric artery

c) Inferior mesenteric artery

d) Celiac trunk

Correct Answer - C
Ans. c. Inferior mesenteric artery
Injury to inferior mesenteric artery is least likely to result in significant
damage among the given options.
Occlusion of the inferior mesenteric artery does not always result in
irreversible ischaemia of the descending and sigmoid colon,
because the marginal artery of the colon usually receives an
adequate supply from the left branch of the middle colic artery.
129. Following road traffic accident, a patient
with vague abdominal pain was
immediately taken to the operation
theatre for emergency laparotomy. On
examination, a large, contained, stable,
non-pulsatile retroperitoneal hematoma
was found on the right side. One-shot
IVU shows a barely discernible
nephrogram on the right side and prompt
uptake and excretion on the left side.
What should be the next step to be
done?
a) Nephrectomy

b) Isolate the proximal renal vessels, open the Gerota's fascia and
explore the kidney

c) Perform on table retrograde pyelography

d) Perform on table angiography

Correct Answer - B
Ans. b. Isolate the proximal renal vessels, open the Gerota's
fascia and explore the kidney
-'With I7/e p281-286; Campbell 10th/1169-1178; Bailey 26th/1286-
1288, 25th/1290-1291; JT Sturm, .IF Perry. Jr and A S Cass. Renal
artery and vein injury following blunt trauma.Annals of surgery.Vol.
182 (6: 696-698.)
130. With respect to peripheral vascular
disease, all of the following statements
are true except:
a) Ankle brachial index < 0.5 indicates critical limb ischemia

b) The ankle-brachial index is different at rest than that during


exercise

c) L-arginine is useful for providing endothelium-independent


vasorelaxation

d) Smoking has a greater association with peripheral vascular


disease than coronary vascular disease

Correct Answer - D
Ans. d. Smoking has a greater association with peripheral
vascular disease than coronary vascular disease > c. L-arginine
is useful for providing endothelium-independent vasorelaxation
Sainston I9/e p/ 730; Schwartz 9/e p751, Bailey 26th/S75, 25th/900-
902)
Smoking has greater association with coronary artery disease
than peripheral vascular disease. L-Arginine is a precursor of Nitric
oxide so helps in vasodilation due to synthesis of
NO by endothelium. Hence it is endothelium-dependent.
131. A surgeon with less experience of
laparoscopic cholecystectomy while
doing laparoscopic surgery found some
stone in common bile duct. What should
he ideally do?
a) Open cholecystectomy with choledochoduodeno​stomy

b) Laparoscopic exploration of CBD and removal of stone

c) Laparoscopic CBD exploration through the cystic duct

d) Open CBD exploration

Correct Answer - D
Ans. d. Open CBD exploration
Laparoscopic CBD exploration is an advanced surgery and if a
surgeon is not trained for this then he should convert the case and
do open cholecystectomy and open CBD exploration.
The preparation and indications for cholecystectomy are the same
whether it is performed by laparoscopy or by open techniques.
For patients in whom a laparoscopic approach is not indicated or in
whom conversion from a laparoscopic approach is required an open
cholecystectomy is performed.
132. Most common post operative
complication of ileo anal pouch
anastomosis in ulcerative colitis is:
a) Pouchitis

b) Pelvic abscess

c) Small bowel obstruction

d) Perianal complication

Correct Answer - A
Ans is 'a' i.e. Pouchitis
"Pouchitis occurs in nearly 50% of patients, and small bowel
obstruction is not uncommon."- Schwartz 9/e
Maingot's 11/e lists results of a meta-analysis of over 8,300 patients
from 18 major studies showing the major complications and
outcomes after ileal pouch–anal anastomosis. According to this
meta analysis:
Most common complication is---- Pouchitis
Next most common is--------------- Small bowel obstruction
In Beal Pouch Anal Anastomosis (Restorative Proctocolectomy) the
entire colon and rectum are resected, but the anal sphincter muscles
and a variable portion of the distal anal canal are preserved. Bowel
continuity is restored by anastomosis of an ileal reservoir to the anal
canal.
The neorectum is made by anastomosis of the terminal ileum
aligned in a "J," "S," or "W" configuration. Most common
configuration used is the "J" configuration as it is the simplest to
construct and functional outcomes are similar in all. (Ref: Schwartz
9/e p1027)
Indications are conditions where colectomy is to be done such as
Ulcerative colitis, FAP, Gardner's syndrome, hereditary nonpolyposis
colon cancer.
Complications:
Short-term complications or those that occur within 30 days of
surgery include pelvic sepsis and abscesses primarily due to
anastomotic or pouch leakage.
Long-term complications include increased bowel movements
(average 6 per day), small bowel obstruction, anastomotic strictures,
fistulas, sexual dysfunction, pouchitis, and adhesions.
133. Which of the following is not a liver
capsular plate?
a) Portal plate

b) Hilar plate

c) Umbilical plate

d) Cystic plate

Correct Answer - A
Ans is 'a' i.e. Portal plate
To surgically manage hilar bile duct carcinoma successfully, it is
important to be familiar with the principal anatomical variations of the
biliary and vascular components of the plate system in the hepatic
hilar area, because all the variations in the bile ducts and vessels
occur in the plate system.
Plate system:
The plate system consists of bile ducts and blood vessels
surrounded by a sheath. There are three plates in the hilar area: the
hilar plate, the cystic plate, and the umbilical plate. The bile duct and
blood vessel branches penetrate the plate system and form
Glisson’s capsule in all segments of the liver, except for the medial
segment.
It is important due to the hilar plate is the most important plate in
liver surgery.
134. Which of the following is not a
prognostic factor for Acute Pancreatitis
a) Serum Amylase

b) Serum Calcium

c) Serum Glucose

d) Serum AST

Correct Answer - A
Answer is A (Serum Amylase) :
Serum Amylase does not form any criteria for prognosis in Acute
Pancreatitis.
Although elevated serum amylase level is important for establishing
diagnosis of acute pancreatitis, it plays no role in predicting
prognosis or severity.
"There appears to be no definite correlation between severity of
pancreatitis and the degree of serum amylase elevation. After 48 to
72 hours, even with continuing evidence of pancreatitis, total serum
amylase levels tend to return to normal." - Harrison
Hyperglycemia (Glucose), Hypocalcemia (Calcium) and elevated
serum AST are all poor prognostic factors in accordance with
Ranson's criteria as elaborated in the previous question.
135. A patient with primary sclerosing cholangitis developed
cholangiocarcinoma. Which is the MOST common site of
cholangiocarcinoma?

a) Distal biliary duct

b) Hilum

c) Intrahepatic duct

d) All

Correct Answer - B
Answer-
Cholangiocarcinoma (CCC):
CCC typically refers to mucin-producing adenocarcinomas (different from HCC) that arise
from the bile ducts.
Anatomically, tumors involving the biliary confluence (hilar cholangiocarcinoma or Klatskin
tumors) account for 60% of cases, with the remainder involving the distal bile duct (20-30%)
or intrahepatic ducts (10-20%).
Also know:
Nodular tumors arising at the bifurcation of the common bile duct are called Klatskin
tumors and are often associated with a collapsed gallbladder.

136. Regular drinking of which of the
following fruit juices can prevent UTI:
a) Raspberry

b) Grape

c) Cranberry

d) Orange

Correct Answer - C
Ans. is 'c' i.e. Cranberry
Cranberry juice has been traditionally used for the treatment and
prophylaxis of urinary tract infection.
Recent randomized controlled trials have demonstrated its utility in
prophylaxis of UTIs but not for treatment.
137. Which among the following is not a
ultrasound feature of Congenital
Hypertrophic Pyloric Stenosis?
a) 95% sensitivity by ultrasound

b) Thickness of pylorus > 4mm

c) Canal length > 16mm

d) High gastric residue

Correct Answer - D
Ans is 'd' i.e. High gastric residue
Ultrasound is the investigation of choice. It can diagnose HPS
accurately in 95% of patients. Criteria for ultrasound diagnosis of
HPS:
a. channel length of > 16mm
b. pyloric thickness of > 4mm
"The diagnosis of pyloric stenosis usually can be made on physical
examination by palpation of the typical "olive" in the right upper
quadrant and the presence of visible gastric waves on the abdomen.
When the olive cannot be palpated, ultrasonography can diagnose
the condition accurately in 95% of patients. Criteria for
ultrasonographic diagnosis include a channel length of >16 mm and
pyloric thickness of >4 mm. "- Schwartz
138. Investigation of choice for diagnosis of
Zenker's diverticulum is:
a) CECT

b) Endoscopy

c) Esophageal manometry

d) Barium swallow

Correct Answer - D
Ans. d. Barium swallow
Investigation of choice for diagnosis of Zenker's diverticulum is a
barium swallow.
It is not a true esophageal diverticula, as it rises above the upper
esophageal sphincter (the cricopharyngeal sphincter)
Usually seen in patients over 50 years a
MC symptom is dysphagia.
Undigested food is regurgitated into the mouth, especially when the
patient is in the recumbent position.
Swelling of the neck, gurgling noise after eating, halitosis, and a
sour metallic taste in the mouth are common symptoms
Cervical webs are seen associated with 50% of patients with
Zenker's diverticulum, which can cause dysphagia postoperatively if
not treated.
139. Which structure lies immediately lateral
to femoral hernia?
a) Lateral cutaneous nerve of thigh

b) Femoral nerve

c) Femoral artery

d) Femoral vein

Correct Answer - D
D i.e. Femoral vein
- The femoral ring refers to the upper opening of the most medial
compartment of the femoral sheath (femoral canal) containing lymph
nodes of Cloquet or Rossenmuller & lymphaticsQ. The femoral
artery occupies the lateral compartment and is separated from the
femoral ring by an intermediate compartment, occupied by the
femoral vein.
Femoral ring, an upper opening in femoral canal (ie medial most
compartment of femoral sheath) is a potentially weak point in lower
abdomen and is the site for femoral hernia. So the intermediate
compartment containing femoral vein lies lateral to the sac of
femoral hernia.
140. Which of the following is not elevated in
a child presenting with jaundice, icterus,
pruritus and clay-colored stools?
a) 5' nucleotidase

b) Alkaline phosphatase

c) Glutamate dehydrogenase

d) Gamma glutamyl transpeptidase

Correct Answer - C
Ans. C. Glutamate dehydrogenase
Glutamate dehydrogenase is not elevated in obstructive jaundice.
Liver diseases in which necrosis of hepatocytes is the predominant
event, such as toxic liver damage or hypoxic liver disease, are
characterized by high serum GLDH levels.
GLDH is important for distinguishing between acute viral hepatitis
and acute toxic liver necrosis or acute hypoxic liver disease,
particularly.
Gamma-glutamyl transpeptidase is important to differentiate
between neonatal hepatitis and biliary atresia in an infant with
jaundice.
141. A 7-day old infant has a leaky
meningomyelocele. The most useful test
for diagnosis and management of the
condition is -
a) Blood-culture and sensitivity

b) Urine-culture and sensitivity

c) Rectal swab-culture and sensitivity

d) Wound swab-culture and sensitivity

Correct Answer - A
Ans is 'a' i.e., Blood-culture and sensitivity
* Most cases of meningocele are recommended for treatment as
soon after birth as possible.
* In the case of a sac which is leaking fluid, the treatment is most
urgent.
* Generally, treatment includes a surgical procedure to close and
remove the soft tissue covering of back.
* As there are chances of bacterial meningitis and sepsis due to
CSF leak, CSF and blood sample should be send for
culture and sensitivity and peri-operative antibiotics (broad
spectrum) can be changed to the specific antibiotics after
the blood and CCF culture and sensitivity report.
142.
Which among the following is an important fatty acid present in breast milk
essential for growth?

a) Docosahexaenoic acid

b) Palmitic acid

c) Linoleic acid

d) Linolenic acid

Correct Answer - A
Docosahexaenoic acid and eicosapentaenoic acid are omega 3 fatty acids which are
important constituents of the communication membranes of the brain, and they are
necessary for normal brain development. They are also active in the retina of the eye.

Docosahexaenoic acid is formed from linolenic acid.
Linolenic acid is an essential fatty acid present in the breast milk.
It is the starting material for the synthesis of eicosapentaenoic acid and Docosahexaenoic
acid.

Biochemically important omega 3 and 6 fatty acids:
Omega 3 acid Omega 6 acid

Linolenic acid Linoleic acid

Eicosapentaenoic acid Arachidonic acid

Docosahexaenoic acid

Ref: Organic and Biological Chemistry By H. Stephen Stoker page 301
143. NESTROFT test is a screeing test for-
a) β -thalassemia

b) Hereditary spherocytosis

c) Autoimmune hemolytic anemia

d) Megaloblastic anemia

Correct Answer - A
Ans. is 'a' i.e., Thalassemia
* NESTROFT ( Naked Eye Single Tube Red Cell Osmotic Fragility
Test) is a screening test for detection of beta thalassemia trait.
Thalassemia screening
* Widespead prevlence of thalessemia has led to a pressing
demand for community screening.
* Various methods for screening of thalassemia trait are available
which include peripheral smear examination, RBC indices,
Meintzer's fraction, discriminant functions, NESTROFT etc.
* NESTROFT TEST is used in many centres in India for screening
of thalassemia trait.
* If mother is NESTROFT positive, the confirmatory test for HbA2 is
done for mother and the father is subjected to
screening by NESTROFT. If father is also NESTROFT positive,
confirmatory test for HbA2 is done for father.
* If both the parents have thalessemia trait, there is 1:4 chance of
fetus having thalassemia major.
* Therefore, prenatal diagnosis is done by chorionic villus sampling
(CVS) in first trimester.
* If CVS confirms the fetus to be having thalasemia major,
termination of pregnancy is indicated after counseling the parents.
* If the fetus has thalassemia minor (trait) pregnancy is continued as
such and baby will have normal lifespan.
* It is worth noting here that a positive NESTROFT test is seen in
other conditions beside beta thalassemia trait.
These are iron deficiency anemia, alpha thalassemia trait,
homozygous and heterozygous HbE, HbS as well as hereditary
persistence of fetal hemoglobin.
Therefore, a positive NESTROFT test should always be followed by
a confirmatory test for HBA2 ( eg. serum electrophoresis).
144. A neonate presents with congestive
heart failure, on examination enlarging
fontanelIae, bruit on auscultation, on
USG shows midline hypoechoeic lesion,
most likely diagnosis:
a) Malformation of vein of galen

b) Aqueduct stenosis

c) Arachnoid cyst

d) Medulloblastoma.

Correct Answer - A
Ans. Malformation of vein of galen
145. Which of the following conditions is
worsened by prostaglandin E infusion?
a) Pulmonary atresia without VSD

b) Hypoplastic left heart syndrome

c) Obstructive TAPVC

d) Aortic arch interruption

Correct Answer - C
Ans. c.'Obstructive TAPVC { Rc/: Ll lllohttn Retlth,. Cunliut. Srr
('unliut surgen''4,,uar oJ rht' se,tinurs in Trnrucic and Cardiovascular
Surgery 2003;4:271-276)
* Obstructive TAPVC is worsened by prostaglandin E infusion
* ln infants with or who have a clinical suspicion for a ductal
dependent congenital heart defect, prostaglandin E, should be
administered until a definitive diagnosis or treatment is
established.
TAPVC
* Total anomalous pulmonary venous connection (TAPVC) is
characterized by abnormal drainage of pulmonary veins into the right
heart either by direct connection into the right atrium or into its
tributaries.
* According to the site or level of connection of the pulmonary veins
to the systemic venous system TAPVC has been classified into four
types :-Infracardiac type of TAPVC is always obstructive whereas
cardiac and supracardiac type may be obstructive or nonobstructive.
- Type I (most common: 45%) : Anomalous connection at
supracardiac level (PV drains into left innominate vein or SVC)
- Type II (25%) : Anomalous connection at cardiac level (PV joins
the coronary sinus or enter right atrium directly).
- Type III (25%) : Anomalous connection at infracardiac level (PV
drain into portal vein).
- Type IV (5%) : Anomalous connection at multiple levels.
X-ray findings of TAPVC
- Cardiomegaly
- Plethoric lung fields
- Snowman or figure of '8' configuration - In supracardiac TAPVC.
- Ground glass appearance of lung - In obstructive TAPVC.
Clinical manifestations of TAPVC
1. Nonobstructive TAPVC - Patients presents with mild cyanosis and
CHF at 6-8 weeks.
2. Obstructive TAPVC - Paients presents with severe cyanosis and
CHF within first week.
- In supracardiac TAPVC the pulmonary veins join to form a single
trunk (common pulmonary vein) which than drain through
anomalous connection
146. Caudal Regression Syndrome is seen in babies of mothers suffering from
which of the following condition?

a) PIH

b) Anaemia

c) Cardiac disease

d) Gestational diabetes

Correct Answer - D
Caudal Regression syndrome is about 200 times more frequent in patients with diabetes,
than in normal healthy women (general population).
It is the most specific fetal malformation in maternal diabetes.
Ref: Williams Obstetrics 22nd Edition, Page 1177; Pediatric Brain And Spine: An Atlas Of
MRI And Spectroscopy By Leena Ketonen Page 383.
147. A child present with recurrent sinusitis
and recurrent chest infections. Chest X-
ray reveals dextrocardia and situs
invertus. The diagnosis is ?
a) Kartagener's syndrome

b) Good-pasture's syndrome

c) Ehlers-Danlos syndrome

d) William Campbell syndrome

Correct Answer - A
Ans is 'a' i.e., Kartagener's syndrome
* Kartagener's syndrome is a subgroup of primary ciliary dyskinesia.
Primary ciliary dyskinesia:
* It is an autosomal recessive syndrome.
* It is characterized by poorly functioning cilia. There is absence or
shortening of Dynein arms that are responsible for the coordinated
bending of cilia.
* Approximately half of the patients with primary ciliary dyskinesia
have kartagener's syndrome.
148. Short child with low T4 and raised TSH
and swelling of pituitary, what is the
diangosis?
a) Primary hypothyrodism

b) Pituitary tumor

c) TSH Secreting pituitary adenoma

d) TSH resistance

Correct Answer - A
Ans is 'a' i.e., Primary hypothyrodism
* Normally, T4 and T3 exert negative feedback on TSH secretion by
pituitary in two ways:
(i) Block the secretion of TSH by pituitary directly major action
(ii) Block the secretion of TRH by hypothalamus (TRH stimulates
the secretion of TSH by pituitary) o In primary hypothyrodism, T4
and T3 are not produced or produced in low concentration by
thyroid. This results in
abolition of negative feedback on TSH secretion. Thus, there is
elevated TSH and pituitary can become hyperplastic
to produce more TSH.
* In pituitary cause of hypothyroidism (secondary hypothyroidism)
both TSH and Thyroid hormone (T4, T3) are low as pituitary does
not secrete TSH and TSH is the major stimulation for production of
T4 and T3.
About option d
* TSH resistance can also produce same picture i.e. raised TSH and
low T4 with pituitary swelling.
* There is thyroid insensitivity to TSH which results in
hypothyroidism (1T4 and T3). Because of reduced thyroid hormone
hypothyroidism (1T4 and T3). Because of reduced thyroid hormone
feedback, TSH is markedly elevated.
* However, Amongst the given options best option is primary
hypothyrodism because TSH resistance is a very rare condition and
further T4 levels are normal in Mild and moderate (Partial) TSH
resistance.
149. Which of the following is not associated
with Down's syndrome?
a) Trisomy 21

b) Mosaic 21

c) Translocation t (15,21), t (21,21)

d) Deletion of 21

Correct Answer - D
Ans. is 'd' i.e., Deletion of 21
150. A 5-years old male child presents with
episodic anaemia and jaundice since
birth. He is least likely to have which of
the following
a) Hereditary spherocytosis

b) Sickle cell anemia

c) PNH

d) G-6-PD deficiency

Correct Answer - C
Ans is `c' i.e., PNH
* Causes of Jaundice since birth are:
(i) Rh incompatibility (erythroblastosis fetalis)
(ii) ABO incompatibility
(iii) Congenital infections (TORCH)
(iv) Sepsis
(v) Concealed hemorrhage
(vi) Red cell membrane defect (hereditary spherocytosis)
(vii) Red cell enzyme defect (G6PD deficiency)
So, option a & d can cause jaundice since birth.
* In sickle cell anemia, affected infants do not develop symptoms in
the first few months of life because the hemoglobin produced by the
developing fetus (fetal hemoglobin) protects the red blood cells from
sickling. This fetal hemoglobin disappears after 5 month of age so
that by 5 months of age, the sickling of the red blood cells is
prominent and symptoms begin.
* PNH is manfested in adults.
So, both PNH and sickle cell anemia does not cause jaundice since
birth.
* But among these two I would prefer PNH as the answer because it
is manifested in adulthood while the patient in question is a 5-years
old child.
Sickle cell anemia symptoms develop at the age of 5 months and it
is one of the cause of jaundice (en.wikipedia.org).
151. On exposure to cold, a neonate shows all
of the following mechanisms except
a) Shivering

b) Crying and flexion of body like fetus position

c) Cutaneous vasoconstriction

d) Increased production of noradrenaline for breakdown of brown


fat in adipose tissue

Correct Answer - A
Ans. a. Shivering
On exposure to cold and wet environment, the neonate tries to
generate heat by increased activity (crying with agitated movement)
and a sympathetic surge that causes vasoconstriction and non-
shivering thermogenesis in the brown fat. Babies attempt to
conserve heat by peripheral vasoconstriction."
"Brown fat is a well-vascularized, sympathetically innervated lipid
collection located in the axillae, groin, nape of the neck,
interscapular area and perinea! area. Cold stress causes the release
of norepinephrine that uncouples beta-oxidation in fat with resultant
heat generation. Preterm and small-for-gestational age infants have
immature thermogenic response because of scanty brown fat
stores."
Role of Brown Fat in Thermogenesis
A newborn baby is more prone to develop hypothermia because of
large surface area per unit of body weight.
In infants, brown fat is an important site of thermogenesis. It results
in the so-called non-shivering thermogenesis.
Brown fat is located around the adrenal glands, kidney, nape of
neck, interscapular and axillary region.
Metabolism of brown fat leads to heat production.
Blood flowing through the brown fat becomes warm and through
circulation, transfers heat to other parts of the body. This mechanism
oh heat production is known as non-shivering thermogenesise.Q
152. A 5 year-old child presents with non-
utatiening macules and pappules on the
skin. Skin biopsy revealed a perivascular
IgA deposition. Which of the following is
the most likely diagnosis?
a) Henoch Shonlein purpura

b) Wegener's granulomatosis

c) Kawasaki disease

d) Drug-induced vasculitis

Correct Answer - A
Ans. a. Henoch-Shonlein purpura (Ref Harrison 19/e p2100, 18/e
p2797)
Presence of non-blanching palpable purpura (vasculitis of skin) and
colicky abdominal pain (vasculitis of gastrointestinal tract) in a five-
year-old child together with evidence of IgA deposition in immune
complexes suggests the diagnosis of Henoch Scholein Purpura
153. Which one of the following drugs is used
for fetal therapy of congenital adrenal
hyperplasia ?
a) Hydrocortisone

b) Prednisolone

c) Fludrocortisone

d) Dexamethasone

Correct Answer - D
Ans. is 'd' i.e., Dexamethasone
`Recommendations for pregnancy at risk consists of adinistration of
dexamethasome a steroid that readily crosses the plecenta'
154. Which of the following is not true about
newborn assessment -
a) APGAR at 7 min indicates neonatal mortality depression

b) APGAR at 1 min, indicators for neonatal resuscitation

c) Fetus can rapidly washout CO2 through placenta

d) Anaerobic metabolism causes acidemia

Correct Answer - A
Ans is 'a' i.e., APGAR at 7 min indicates neonatal mortality
depression
Later times APGAR score (after 5 minutes) indicates about long
term neurological damage (not neonatal mortality)
Interpretation of APGAR Score
The test is generally done at one and five minutes after birth, and
may be repeated later if the score is and remains low. Scores 3 and
below are generally regarded as critically low, 4 to 6 fairly low, and 7
to I 0 generally normal.
* A low score on the one-minute test may show that the neonate
requires medical attention (e.g. resuscitation) but is not necessarily
an indication that there will be long-term problems, particularly if
there is an improvement by the stage of the five-minute test. If the
Apgar score remains below 3 at later times such as 10, 15 or 30
mintues, there is a risk that the child will suffer longer-term
neurological damage. There is also a small but significant increase
of the risk of cerebral palsy. However, the purpose of the Apgar test
is to determine quickly whether a newborn needs immediate medical
care; it was not designed to make long-term predicitions on a child's
health.
CO2 transport across placenta
* CO2 is cleared by placenta by simple diffusion. CO2 is produced
abundantly in the fetus, and the PCO2 of fetal blood is higher than
maternal blood. CO2 therefore diffuses from fetal blood, through the
placenta, into the maternal circulation, and is disposed by expiration
from mother's lung.
Anaerobic metabolism causes acidemia due to lactate (lactic acid)
production
* Anoxic perfusion causes an increase in glucose consumption
which is more than two fold higher than that seen in the oxygenated
perfusion, resulting finally in placental uptake of glucose not only
from the maternal but also from the fetal circulation.
* Lactate production is increased during the anoxic perfusion, while
the final tissue energy value lies between the values observed for
fresh tissue and for the oxygenated perfusion. The shift to anerobic
metabolism shown by placental tissue in anoxic conditions enables
continued functioning of the tissue over the 2-h perfusion period but
it appear that under anoxic conditions the tissue may incur an
energy debt not observed in oxygenated perfusions.
155. Delayed neuronal migration and
organization leads to certain disorders.
Which of the following is the least likely
possibility?
a) Lissencephaly

b) Schizencephaly

c) Polymicrogyria

d) Focal cortical dyplasia without balloon cells

Correct Answer - D
Ans. d. Focal cortical dyplasia without balloon cells (Ref
http://enm kipedia.org/wiki/Neuronal migration_ disorder)
Delayed neuronal migration and organization leads to certain
disorders. Least likely possibility is focal cortical dyplasia without
balloon cells
156. In patients with osteoarthritis of knee
joint, atrophy occurs most commonly in
which muscle :
a) >Qudriceps only

b) >Hamstrings only

c) >Both (a) and (b)

d) >Gastrocnemius

Correct Answer - A
Quadriceps only Met Maheshwari 3/e, p 253;Apley's 8/e, p 472]
In osteoarthritis of knee joint, the quadriceps muscle is usually
wasted.
157. Osteoporosis is characterized by all the
following except ?
a) Decreased bone mineral density

b) Decreased Serum Calcium, phosphorus and alkaline


phosphatase is seen

c) Glucocorticoids can cause osteoposis

d) Dorsolumbar spine is the most coomon site of osteoporotic


fracture

Correct Answer - B
Ans. is 'b' i.e., Decreased Serum Calcium, phosphorus and alkaline
phosphatase is seen
Osteoporosis is a state of decreased mass per unit volume of a
normally mineralized bone. Osteoporosis is the commonest
metabolic bone disease. Osteoporosis is characterized by an
abnormally low bone mass (low bone density) and defects in bone
structure, a combination of which renders the bone unusually fragile
and at greater than normal risk of fracture. Bone depletion may be
brought about by predominant bone resorption, decreased bone
formation or a combination of the two.
158. In Recurrent Anterior dislocation of
shoulder, the movements that causes
dislocation is
a) Flexion and internal rotation

b) Abduction and external rotation

c) Abduction and internal rotation

d) Extension

Correct Answer - B
B i.e. Abduction & external rotation
159. A cricketer got injured while holding a
catch, following which he complained of
pain over the base of thumb. Which
structure is most likely to be injured.
a) Volar plate

b) Extensor pollicis longus

c) Abductor pillicis longus

d) Ulnar collateral ligament

Correct Answer - D
D i.e. Ulnar collateral ligment
160. A 56-year-old female presents with
nocturnal pain in the right thumb, index
and middle finger for the past 3 months.
All of the following provocative tests can
be performed for the diagnosis of the
condition except :
a) Phalen's test

b) Finkelstein test

c) Tinel's sign

d) Tourniquet test

Correct Answer - B
Ans. b. Finkelstein test (Ref. Maheshwuri 3/e
Nocturnal pain in the right thumb, index and middle finger for the
past 3 months in a 56-year old female is highly suggestive of Carpal
tunnel syndrome. Finkelstein test is used to diagnose DeQuervain's
tenosynovitis, not the carpal tunnel syndrome.
161. Whicn is not a marker of new bone
formation?
a) Alkaline phosphatase

b) Osteocalcin

c) Urine hydroxyproline

d) Pro collagen

Correct Answer - C
C i.e. Urine hydroxyl proline
162. Which is not a deep heat therapy
a) Short wave diathermy

b) Ultrasound therapy

c) Infrared therapy

d) Microwave therapy

Correct Answer - C
Infrared therapy [Ref. Maheshwari 3/e, p 66; Internet - medicine]
Heat therapy is a form of physiotherapy used in many painful
conditions eg. arthritis, sprains, muscle spasms etc. It is of 2 types
depending on penetration of heat :
- superficial heat therapy
- Deep heat therapy
Superficial heat therapy
Only the skin and subcutaneous tissues are heated
Superficial heating modalities do not heat deep tissues because the
subcutaneous layer of fat beneath the skin surface acts as a thermal
insulator and inhibits heat transfer.
Superficial heating modalities include :
- hot water bottle -
chemical packs
warm bath - paraffin
wax bath
- hot packs (eg. Kenny pack) - moist
air cabinet
- hot soaks or compresses -
infrared lamp
Deep heat therapy
Deeper structures (including muscles) are heated
In general, the transfer of heat is of three types - conduction,
convection & conversion (or radiation) [We have read these during
our primary schooling]
Whereas superficial heating modalities use all three modes of
transfer of heat; deep heating modalities use only conversion or
radiation method to heat the deep tissues.
Deep heating modalities are
a. Short wave diathermy
- heat generated by high frequency alternating current using a short-
wave diathermy emitter.
b.Microwave diathermy
- this uses electromagnetic radiation energy to heat the deep tissues
c. Ultrasound therapy
- uses high frequency sound energy
163. Gallows traction is used in management
of fracture shaft ?
a) Femur

b) Tibia

c) Humerus

d) Ulna

Correct Answer - A
Ans. is 'a' i.e., Femur
Gallows traction is used for treatment of fracture shaft femur, in
infants and children < 2 yrs of age.
Fracture shaft femur in infant and young children
It is published that upto one half of infants and young children who
sustain femur fractures are victims of child abuse and child abuse
occurs in 50 - 80% of children (< 2 years of age) with femoral
fracture.
Other causes of fracture are osteogenesis imperfecta (blue sclera,
hearing loss, multiple fracture), bone tumors (eosinophilic
granuloma, aneurysmal bone cyst).
Fracture heals within 2 weeks in infant and children upto 2 years of
age.
Treatment depends upon the age of the patient : ?
1) Conservative : - It is the treatment of choice in children.
i. 0-2 yeas :- Plaster spica or modified Bryant or Gallow's traction or
pavllic harness (< 6 month of age).
ii. 2-10 years : - Split Russel traction
iii. 10-15 years : - 90-90° femoral skeletal traction.
2) Surgical
Surgery is done less commonly in children.
It is indicated in older children when closed reduction by
conservative treatment is not possible.
Intramedullar titanium elastic nailing is the surgery of choice.
164. A middle aged lady presents with
complaints of lower back pain. ON
examination there is weakness of
extension of right great toe with no
sensory impairment. An MRI of the
lumbosacral spine would most probably
reveal a prolapsed intervertebral disc at
what level?
a) L3 - L4

b) L4-L5

c) L5-S1

d) S1-S2

Correct Answer - B
B i.e. L4 - L5
165. Patient develops myelopathy post
trauma. What dose of methyl
prednisolone is to be given:
a) 30 mg/kg within 3 hrs

b) 45 mg/kg within 6 hrs

c) 60 mg/kg within 9 hrs

d) 75 mg/kg within 12 hrs

Correct Answer - A
A i.e. 30mg/kg within 3 hrs 38.
166. Calcification of the intervertebral disc is
present in:
a) Maple syrup urine disease

b) Homocystinuria

c) Ankylosing spondylitis

d) Achondroplasia

Correct Answer - C
Ans. Ankylosing spondylitis
167. All of the following puy NiUiugiLai
changes are seen in pregnancy except:
a) Increased stroke volume

b) Increased cardiac output

c) Increased intravascular volume

d) Increased peripheral resistance

Correct Answer - D
Ans. d. Increased peripheral resistance
(REF-Dutta6/e pg 53,51)
During the physiological course of pregnancy, maternal peripheral
vascular resistance decreases.
"Pregnancy is a state of hypervolemia. There is active retention of
sodium (900 mmol), potassium (350 mmol) and water."- Dutta 6/e
p51
168. Primary Amenorrhea with absent uterus, normal breasts and scanty pubic
hair is seen in:

a) Mayer Rokitanski Kuster Hauser syndrome

b) Turner's syndrome

c) Androgen Insensitivity Syndrome

d) Noonan syndrome

Correct Answer - C
Primary Amenorrhea with absent uterus, normal breasts and scant pubic hair is consistent
with a diagnosis of Androgen Insensitivity Syndrome.

Androgen Insensitivity Syndrome is caused by inability of end organs to respond to


androgens.
In this either the androgen receptor is deficient or the receptor is defective. Karyotype is
46,XY.
These individuals are phenotypically females with breast development, normal female
external genitalia, blind ending vagina, absent uterus, tubes, cervix and upper 2/3 rd
vagina, scanty pubic hair, with gonads in the labia or inguinal canal.

Ref: William's Gynecology, 1st Edition, Page 368 ; Speroff, 7th Edition, Pages 420, 421,
422 ; Textbook of Gynecology By DC Dutta, 4th Edition, Pages 405-6 ; Pediatric
Endocrinology : A Practical Clinical Guide By Sally Radovick, Margaret H. MacGillivray,
Page 389
169. Normal functioning ovaries are seen in
which of the following condition?
a) Rokitansky-Kuster-Hauser syndrome

b) Turner syndrome

c) Sawyer syndrome

d) Adnrogen insensitivity syndrome

Correct Answer - A
Ans. a. Rokitansky-Kuster-Hauser syndrome IR,,f Shalt c Giner
14th/156-151 Vovaks Gynecology 14/e p1036)Normal functioning
ovaries are seen in Rokitansky-Kuster-Hauser syndrome
170. After coming head of breech will have
difficulty in delivery in all of the following
conditions except :
a) Hydrocephalus

b) Placenta previa

c) Incomplete dilation of cervix

d) Extension of head

Correct Answer - B
Ans. is b i.e. Placenta previa
Entrapment of the after coming head occurs in case of:
Incompletely dilated cervix
Hydrocephalus
Extended head I deflexed head
Contracted pelvis
Management :
If entrapment occurs due to incompletely dialated cervix Vuhrssen's
incisions' are placed over the cervix avoiding the 3 and 9'0 clock
position.
Intravenous nitroglycerine can be used to relax the cervix.
Replacement of the fetus higher in to the vagina and uterus,
followed by cesarean delivery (Zavanelli maneouvre)
Symphysiotomy used to widen the anterior pelvis is practiced in
some countries and is associated with good infant outcomes.
Also Know
Impacted Breech
Inspite of good uterine contractions and complete dilatation of the
cervix, the breech fails to descend.
This occurs only in extended breech and is usually due to
disproportion.
Impaction can occur at the inlet, cavity or outlet.
If within 30 min of full cervical dilatation the breech does not
descend and distend the perineum, cesarean section is done
regardless of the level of impaction.
171. Which of the following is the least likely
complication of pregnancy-induced
hypertension?
a) Renal failure

b) Pre-eclampsia

c) HELLP syndrome

d) Fetal macrosomia

Correct Answer - D
Ans. d. Fetal macrosomia (Ref- Dutta6/e , pg227)
Fetal macrosomia is the least likely complication of pregnancy-
induced hypertension among the options provided.
172. Maternal to child transmission of HIV is
prevented by -
a) Nevirapine

b) Lamivudine

c) Didanosine

d) Abacavir

Correct Answer - A
Ans. is 'a' i.e., Nivirapine
. Single dose of nivirapine given to the mother at the onset of labor
followed by a single dose to the newborn within 72 hours of birth
decreased transmission by 50%. This is the preferred regimen now
in developing countries.
173. What would be the ideal management of
a woman with infertilty who is detected
to have bilateral cornual block on
hysterosalpingography?
a) Tuboplasty

b) In-vitro fertilization

c) Hydrotubation

d) Hysteroscopy and laparoscopy

Correct Answer - D
Ans. d. Hysteroscopy and laparoscopy (Ref The Infertility
Manual/p266-267; Practical approach to infertility management by
aiilr Rcensal/p33:
Hysteroscopy and laparoscopy would be the ideal management of a
woman with infertility who is detected to have bilateral cornual block
on hysterosalpingography.
174. A 20 year old female is diagnosed with
granulosa cell tumor of the ovary. Which
of the following biomarkers would be
most useful for follow-up of patient?
a) CA 19-9

b) CA 50

c) Inhibin

d) Neuron-specific -enolase

Correct Answer - C
Ans. is 'c' i.e., Inhibin
o Granulosa call tumor is positive for vimentin, inhibin, CD99
175. HRT is helpful in all of the following
except
a) Vaginal atrophy

b) Flushing

c) Osteoporosis

d) Coronary heart disease

Correct Answer - D
Ans. is d i.e. Coronary heart disease
Friends, Harrison 16/e. p 30 breaks a popular myth of using HRT for
prevention of Coronary. Heart Disease. Until recently, it was
believed that the sex specific effect of Gonadal Steroids on CVS and
lipid metabolism accounted for the different rates of Coronary heart
disease in women as compared to men.
Estrogen increases HDL and decreases LDL whereas androgens
have the opposite effect and this was further supported by the
increase in incidence of Coronary heart disease after menopause.
These findings led to the widespread use of HRT for primary and
secondary prevention of Coronary heart disease. (CHD)
But recent trials have shown an increase in the incidence of CHD in
women placed on HRT as compared to those not on HRT.

This fact is supported by Williams Gynae. 1/e, p 494 which says
"In the many reviews and discussions following WHI (Women Health
Initiative), most clinicians agree that Hormone Therapy is associated
with an increased risk of CHD in older menopausal women and an
increased risk of breast cancer, stroke, venous thromboembolism
and cholecystitis."
Advantages of oestrogen in HRT :
Prevents osteoporosis° (Greatest potential benefit of oestrogen
therapy)
Decreases hot flushes°
Vaginal application of oestrogen decreases vaginal atrophy°
Hazards of Prolonged estrogen use :
Risk of Endometrial hyperplasia and Ca endometrium increases°
Risk of breast Cancer increases°
T ed Risk of Gall bladder disease°
Ted Risk of Thromboembolic events°
Friends, here I would also like you to know - Harrison 16/e, p 30
gives a table on the potential benefits and harms
176. Placental alkaline phosphatase is marker
of:
a) Theca cell tumor

b) Teratoma

c) Choriocarcinoma

d) Dysgerminoma

Correct Answer - D
Ans. is d i.e. Dysgerminoma
Placental alkaline phosphatase and LDH are tumour markers of
dysgerminoma.
177. For an elect , forceps deliver) tth a
cephalic presentation of the fetus, which
of the following is correct?
a) Fetal head should be at station '0'

b) Forceps can be used for >15 degree head rotation from AP


diameter

c) Can be used in vertex, mentoanterior and face presentation

d) Should not be used with caput succedaeneum

Correct Answer - C
Ans. c. Can be used in vertex, mentoanterior and face presentation
(Ref. Dutta 7/e p573, 575: Williams 23/e p513)
Elective forceps delivery with a cephalic presentation of the fetus
can be used in vertex, mentoanterior and face presentation.
"Presence of caput succedaneum is not a contraindication for
forceps application."
"Forceps is applied wizen head reaches the perineum (not at zero
station)." "Forceps can be applied till maximum 45° rotation."
178. Karyotyping of the fetus can be done
through all of the following invasive
methods except:
a) Chorionic villus sampling

b) Cordocentesis

c) Amniocentesis

d) Fetal skin biopsy

Correct Answer - D
Ans. d. Fetal skin biopsy (Ref: Williams 23/e p299-301; Dutta 6/e
p107, 108, 642)
"Currently there are three routine ways of obtaining fetal tissue:
Amniocentesis at 15 to 20 weeks, gestation, chorionic villas
sampling at 10 to 12 weeks, and percutaneous umbilical blood
sampling (cordocentesis) past 18 weeks
179. Which of the following histories is not an
indication to perform oral glucose
tolerance test to diagnose gestational
diabetes mellitus?
a) Past history of eclampsia

b) Past history of congenital anomalies in the fetus

c) Past history of unexplained fetal loss

d) Past history of polyhydramnios

Correct Answer - A
Ans. a. Past history of eclampsia (Ref Williams 23/e p1107. Dutta
7/e p281, 6/c p284)
Past history of eclampsia is not an indication to perform oral glucose
tolerance test to diagnose gestational diabetes mellitus.
180. An infant presented with erythematous
lesions on cheek, extensor aspect of
limbs, mother has history of bronchial
asthma, the probable diagnosis is
a) Air borne contact dermatitis

b) Atopic dermatitis

c) Seborraehic dermatitis

d) Infectious eczematoid dermatitis

Correct Answer - B
B i.e. Atopic Dermatitis
181. Intra epidermal blisters are seen in:
a) Bullous pemphigoid

b) Pemphigus folliaceous

c) Dermatitis herpeteformis

d) Bullous SLE

Correct Answer - B
B i.e. Pemphigus folliaceous
182. All are true about thymus swelling
except:
a) Widening of mediastinum on X-Ray

b) Sharp border with shail like appearance

c) Steroid administration reduces size of swelling

d) Shift of trachea on X-ray

Correct Answer - D
D i.e. Shift of trachea on X Ray
Prominent but normal thymus is the most common pseudotumor of
anterior mediastinum. On imaging anterior
mediastinal tumors displace the trachea and oesophagus posteriorly
and laterlly in a contradistinction to a normal enlarged thymus which
does not displace adjacent structuresQ
Thymic Tumors
Normal Thymus Gland
It is the commonest cause of mediastinal abnormality in infants and
is usually seen as triangular soft tissue mass projecting to one
side (usually right) of mediastinum.
It may disappear during severe neonatal infections, or after major
surgery corticosteroid treatmentQ, but may reappear following
recovery from illness.
Thymus gland is completely absent in Di-George's syndrome, an
immune deficiency disease involving T lymphocyes. It is seen as a
triangular arrow head or bibbed structure in children and young adult
patients on CT but undergoes fatty involution in adults & elderly.
Enlargement of thymus gland can be d/t thymoma,
thymic hyperplasia, thymic carcinoma, lymphoma, thymo lipoma,
carcinoids & germ cell tumors & thymic cysts.
Thymic Hyperplasia
Although uncommn & rare, it is the most common anterior
mediastinal mass in paeditric age group.
The most common association of thymic hyperplasia is mysthenia
gravis (65%) and thyrotoxicosis (eg Grave's disease, treatment of
hypothyroidism etc. Other associations include Addison's disease,
acromegaly, SLE, RA, and after stress atrophy, where the thymus
gland initially atrophies in patients on chemotherapy, or
corticosteroid treament, irradiation, stress, severe illness (burns) or
after treatment for Cushing's disorder and then becomes larger than
its previous normal size (i.e. enlarges) once the treatment is stopped
or stress is ended. The phenomenon is called rebound thymic
hyperplasia. It is differentiated from recurrent malignant disease on
the basis of a known reason for rebound and presence of normal
shaped enlarged thymus
Rarely cause visible enlargement, but when it does, both lobes are
enlarged, usually uniformly with a diffuse symmetrical
enlargment. Radiological signs include.
1. Wave sign is a rippled thymic contour (border) d/t indentation
by anterior rib ends.
2. Notch sign is indentation at the junction of thymus with heart.
3. Sail sign is a triangular density projection from superior
mediastinum on one or both sides.
4. Shape change with respiration & position
Thymoma
It is the most common tumor of thymus in adults and most common
primary tumor of anterior mdidastinum in adultsQ.
Most (90%) thymomas arise in upper anterior
mediastinum usually anterior to ascending aorta, lying on the right
ventricle outflow tract and pulmonary artery.
It is rare under 20, extremely unusual below 15 and usually presents
at 50 year age (earlier in those who present with mysthenia gravis)
Thymoma is often asymptomatic 50% but can also present with
mysthenia; red cell aplasia, hypogammaglobulinemia. About 10-25%
mysthenics have thymoma and 25-50% of thymoma patients have
mysthenia gravis.
It may cause mediastinal wideningQ and displacement of heart and
great vessels posteriorly (but not trachea)-in benign cases; and can
invade mediastinal fat & pleura in invasive malignant variety.
It may be undetectable on chest x-ray when small, indicating the
need of CT (which is investigation of choice & most sensitive).
Thymomas give rise to asymmetrical focal swelling which appear as
well defined round or oval soft tissue mass projecting to one side of
anterior mediastinum. It may contain punctate or curvilinear
calcification and areas of low attenuation d/t cystic degenration. MRI
is also useful for diagnosis.
Thymic Carcinoid
It presents with cushing's syndrome (d/t ACTH serection),
hyperparathyroidism and inappropriate ADH secretion. Radiographic
& CT features resemble thymoma
Thymolipoma
Can grow to a very large size before discovery and,being soft, mould
themselves to the adjacent mediastinum and diaphragm, mimicing
cardiomegaly or lobar collapse.
Very large soft tissue mass with less radiographic density than
expected for its size, which alters it shape on respiration indicates
thymolipoma
183. Only definitive indication of systemic
steroids in psoriasis is
a) Pustular psoriasis

b) Erythroderma

c) Psoriatic arthropathy

d) Impetigo herpetiformis

Correct Answer - D
D i.e. Impetigo herpetiformis
- Systemic steroids should be avoided in routine care of psoriasis Q
because the disease usually breaks through, requiring progressively
higher doses to control symptoms and withdrawl of drug is usually
associated with frequent relapse in form of life threatening
erythrodermic psoriasis (with exfoliative dermatitis) and pustular
psoriasis (with pus lakes)Q.
- However, systemic steroids may have a role in management of
1. Persistent, otherwise uncontrollable (e.g. with metabolic
complications), psoriatic erythroderma and in fulminant generalized
pustular psoriasis (von Zumbusch type) if other drugs are ineffective
or contraindicatedQ.
2. Pustular psoriasis in pregnancy is called impetigo herpatiformis. In
pregnancy, safest drug for treatment of pustular psoriasis is
prednisoloneQ.
3. Severe psoriatic polyarthritis threatening severe irreversible joint
damageQ.
Pustular psoriasis may develop after strong topical or systemic
steroids have been used and then abruptly withdrawnQ. It presents
with development of generalized pustules all over the bodyQ.
Erythematous skin rash with multiple pus lakes suggests a diagnosis
of generalized pustular psoriasis. As retinoids, methotrexate &
PUVA cannot be used (or contraindicated) in pregnancy.
Corticosteroids form the mainstay of treatment for generalized
pustular psoriasis in pregnancyQ. Localized disease is best treated
with topical steroid & fulminating generalized disease is best treated
with systemic prednisolone (oral).
184. Painful vaginal ulcer with inguinal
lymphadenopathy and school of fish
appearance of microorganism or
microscopy are characteristic of:
a) Syphilis

b) LGV

c) Granuloma inguinale

d) Chancroid

Correct Answer - D
Ans. is. 'd' i. e., Chancroid
Chancroid is a bacterial sexually transmitted disease (STD) caused
by infection with Haemophilus ducreyi.
It is characterized by painful necrotizing genital ulcers that may be
accompanied by inguinal lymphadenopathy. It is a highly contagious
but curable disease.
H ducreyi, a small, gram-negative, facultative anaerobic bacillus that
is highly infective.
It is pathogenic only in humans, with no intermediary environmental
or animal host.
H ducreyi enters the skin through disrupted mucosa and causes a
local inflammatory reaction
H ducreyi is transmitted sexually by direct contact with purulent
lesions and by autoinoculation to nonsexual sites, such as the eye
and skin.
The organism has an incubation period of 1 day to 2 weeks, with a
median time of 5-7 days.
185. A man presents with a 6-month history of
recurrent oral ulceration. He has
yellowish ulcerations on his lip which are
erythematous, with a central halo and
nodular lesions on his shin. Diagnosis
is:
a) Behcet's disease

b) Pemphigus vulgaris

c) Fixed drug eruption

d) Herpes labialis

Correct Answer - A
Ans. a. Behcet's disease (ref- Harrison 19/e Pg 2194, 18/e Pg 2801)
Diagnosis of a patient who presents with a 6-month history of
recurrent oral ulceration, yellowish ulcerations on the lip,
erythematous, with a central halo and nodular lesions on shin is
Behcet's disease.
Associated with HLA-BS9
Disease affect young patients; Both males and females are
equally affected
186. Which of the following can lower EEG
amplitude
a) N20

b) Ketamine

c) Hypothermia

d) Early hypoxia

Correct Answer - C
C i.e. Hypothermia
- EEG activation (i.e. high frequency & low voltage activity) is seen
in (Mn-"Small BBC In English Stimulates No Hypoxic Keets") i.e.
Barbiturates (small dose), Benzodiazepines (small dose), Mild
hypercapnia (TCO2), Inhalational agents (subanesthetic), Etomidate
(small doses), sensory stimulation, N20, Hypoxia (early), ketamine.
Here small = small, mild, early or subanesthetic dose.
EEG depression is caused by (Mn- Marked Hypo) i.e. hypothermia,
hypocapnia, Propofol and Opioids and all marked (i.e. marked
hypoxia (late), marked hypercapnia, large dose of barbiturates,
etomidate & inhalational agents (1-2 MAC).
187. Which of the following is not true about
xenon anesthesia
a) Non explosive

b) Minimal cardiovascular side effects

c) Slow induction and slow recovery

d) Inhibits Ca.' pump and low blood gas solubility

Correct Answer - C
C i.e. Slow induction and recovery
188. Epidermoids can be differentiated from
arachnoid cyst on MRI by:
a) Contrast enhancement

b) Smooth margins

c) Restricted diffusion

d) CSF signal on FLAIR

Correct Answer - C
C i.e. Restricted diffusion
Epidermoids can be differentiated from arachnoid cyst
characteristically by hyper-intense diffusion weighted images
because of restricted diffusion of epidermoid cystQ. FLAIR and
proton images are less specific than DWI.
189. A patient is suspected to have vestibular
schwannoma the investigation of choice
for its diagnosis is:
a) Contrast enhanced CT scan

b) Gadolinium enhanced MRI

c) SPECT

d) PET scan

Correct Answer - B
Ans. B (Gadolinium enhanced MRI)
1. Vestibular schwannoma (VS) - arise from inferior vestibular division
of eight nerve as from the auditory division
a. MRI with gadolinium is the current standard for screening for
suspected vestibular schwannomas because of the high false
negative rate associated with BSERA Treatment
2. Under 65 years - small intracranial tumors (within the internal
auditory canal) may be surgically removed through the transmastoid
labyrinthine route
a. Larges tumors (> 3 cm) are associated with increased incidence
of hearing loss, dysequilibrium, headache facial numbness, and
diplopia are removed via a suboccipital craniotomy
b. Huge ones can only be removed via combined suboccipital and
translabyrinthine approach
3. Over age 65 with slowly growing tumors, observation is the proffered
treatment.
190. A middle aged female presents with
slowly progressive weakness of lower
limbs, spasticity and recent onset
hesitancy of Micturition. On neurological
examination there is evidence of dorsal
myelopathy. MRI scan of spine reveals
middorsal intradural contrast enhancing
mass lesion. Diagnosis is:
a) Intradural lipoma

b) Dermoid cyst

c) Meningioma

d) Epidermoid cyst

Correct Answer - C
Ans. c. Meningioma (Ref. Harrison 19/e p602, I 8/e p3388;
Chapman 4/e p431; Sahicton /9/e p 1888- 1889; Schwa/17 9/e
p1540- I 541: Bailey 26/e p614, 25/e p633)
Diagnosis in a middle aged female with slowly progressive
weakness of lower limbs, spasticity and recent onset hesitancy of
micturition with evidence of dorsal myelopathy and middorsal
intradural contrast enhancing mass lesion on MRI is meningioma.
191. Cavernous hemangioma is characterized
by:
a) Reticulated popcorn like configuration

b) Well defined nidus

c) Well defined arterial feeder

d) Phlebectasis

Correct Answer - A
A i.e. Reticulated popcorn like configuration
T2 gradient echo MRI is the investigation of choice for identification
of cavernous angioma (cavernous hemangioma or cavernoma),
which typically demonstrates a mass characterized as popcorn in
appearance (mulberry shaped lesion)Q.
192. Which of the following is the least useful
for diagnosis of Spondylolisthesis?
a) CT

b) MR

c) X-ray lumbar spine- AP view

d) X-ray lumbar spine- Lateral view

Correct Answer - C
Ans. c. X-ray lumbar spine- AP view (Ref Tureks 6/e p503;
Maheshwarl 3e/p 237; Wolfgang 7/e p 228-29, 206; Sutton 7/e p/66'
Although for imaging of spondylolisthesis usually all are done, AP
view appears to be least useful of these choices. On AP view,
spondylolisthesis can show a sign called Napolean Hat sign. The
lateral view is useful in detecting spondylolisthesis; it may
demonstrate the pars defect.
Spondylolisthesis is forward slip of one vertebrae upon another; so
it is best viewed (or seen earliest) in sagittal images of spine i.e.
lateral and oblique X-ray of spine and saggital and axial views of CT
and MRIQ
193. Somatic passivity is a feature of :
a) Paranoid schizophrenia

b) Hypochondriasis

c) Depression

d) Body dismorphic disorder

Correct Answer - A
A i.e. Paranoid schizophrenia
194. Panic attack is associated with a
disturbance in all of the following
neurotransmitters except:
a) Serotonin

b) GABA

c) Glutamate

d) Dopamine, CCK, pentagastrin

Correct Answer - C
C i.e. Glutamate
Panic disorder is associated with noradrenaline (norepinephrine),
cholecystokinin (CCK)- pentagastrin /tetrapeptide (administration or
agonism of both); GABA (antagonism) and serotonin (decrease).
Glutamate studies are either equivocal or in preclinical phase d/t fear
of convulsions.
Neurochemical Aspects of Panic Disorder
Panicogenic Agents
Intravenous sodium lactate or inhalation of 5-35% CO2 can induce
panic attacks in perons with panic disorder while sparing those
without such a history - these substances are
k/a panicogenic. Other examples of panicogenic agents
include caffeine, cholecystokinin 4, noradrenergic agents yohimbine
& isoproterenol, CABA antagonist such as flumezanil, reverse
benzodiazepine agonists such as /3-carbolines (because
benzodiazepine agonists eg alprazolam, clonazepam treat panic
disorders, so drugs reversing their actions precipitate panic attack).
Serotonin
Gorman's neuroanatomical hypothesis states that both panic attack
in humans & conditioned fear responses in animals are similar in
autonomic arousal, fear evoked by specific cues (i.e. contextual fear)
and avoidance of these cues. Both are mediated by fear network
consisting of amygdala & its afferent & efferent projections
particulary its connections with hippocampus, medial prefrontal
y-Amino Butyric Acid
GABA system is certainly involved in panic disorder as evidenced by
1) Benzodiazepine agonist such as alprazolam, &
clonazepam are effective in treatment of panic disorder.
2) Reverse benzodiazepine agonist such as fl-carbolines cause
panic attacks.
3) CABA antagonist such as flumezanil have
increased panicogenic effects.
Cholecystokinin (CCK)
- Cholecystokinin (CCK) is a neuropeptide derived from 112 amino
acid precursor compound which acts via CCK-A (CCK-1) and CCK-
B (CCK-2) receptors. CCK-8 (octapeptide) is most abundant, does
not cross BBB (so induce anxiety only after intracranial
administration) and acts on both receptors CCK4 (tetrapeptide) is
anxiogenic and a primary agonist of CCK-B receptors. Both CCK-4
and CCK-5 (pentagastrin) are panicogenic and effects could be
blocked by CCK antagonist or treatment with GABA agonist such
as vigabatrin or
cortex, hypothalamus & brain stem. SSRIs desensitize the fear
network. SSRIs increase serotonergic transmission in
brain. Serotonergic neurons originate in brain stern raphe & project
throughout CNS and some of these projections have inhibitory
influences. For example the greater the activity in the raphe.
1) The greater the inhibition of noradrenergic neuron in locus cerulus
(resulting in reduction in CVS sysmptoms such as tachycardia).
2) Greater the inhibition in periaqueductal gray region (resulting in
reduction in avoidance behavior).
3) Increased senetonergic activity also reduces hypothalmic
release of corticotropin releasing factor, thereby resulting in a
reduction of cortisol and reduction in activity of locus ceruleus
thereby 1/t reduction in fear.
4) SSRIs may also directly inhibit activity of lateral nucleus of
amygodala.
amygodala.
Noradrenaline
Noradrenaline (norepinephrine) agents yohimbine & isoproterenol
stimulate panic attacks suggesting a possible subsensitivity of
presynaptic a2 inhibitory adrenoreceptors. Both increase firing rate
of locus ceruleus (brain alarm system). Most effective medications in
treatment of panic disorder in fact decrease locus ceruleus firing rate
& most panicogenic stimuli increase the locus ceruleus firing rate.

tiagabine. However, clinical trials of C1-988, a CCK-B antagonist


have failed to abate (treat) anxiety symptoms induced by CCK-4 or
MCPP.
Patients with panic disorder have lower CSF levels of CCK than
healthy controls, although it is unclear whether it is d/t higher CCK
turnover or greater receptor sensitivity.
CCK has its highest levels in the cerebral cortex, hippocampus,
amygdala, caudate & putamen, with intermediate levels in thalamus
& hypothalamus. It is also prevent in gastrointestinal tract. CCK
receptors are found in greatest density in the cortex, hypothalamus,
substantia nigra & PAG.
Glutamate
Glutamate is the primary excitatory neurotransmitter of CNS &
precursor of GABA. It consists of 2 families of receptors:
Metabotropic (m G1uR) & ionotropic receptors. Several preclinical
(animal) studies have shown anxiolytic properties of group 2 & group
3 m-Glu- receptor agonist. However, human studies have failed to
give a result distinct from placebo or were discontinued d/t findings
of convulsions in animal studies of compound. This is
understandable risk, given that glutamate enhancing drugs have
neuro excitatory properties and glutamate inhibitors are used as anti
convulsants. Similarly MG1u-5 and m GIuR7 antagonists and
AMPA/kainate receptor blockers have shown antianxiety properties
in animals.
195. Counter transference is:
a) Type of defence mechanism

b) Psychic connection between patient & disease with transfer of


psychic energy from body parts to brain

c) Implies doctor's feelings towards patient

d) Patient's feelings towrds doctor during psychotherapy

Correct Answer - C
C i.e. Implies doctor's feelings towards patient
Transference implies patient's feelings towards clinician (therapist)
during psychotherapyQ, whereas counter transference implies
clinician's feelings towards the patientQ.
196. Type D personality are recently found to
be at risk of developing:
a) Coronary artery disease

b) Depression

c) Schizophrenia

d) Mania

Correct Answer - A
A i.e. Coronary artery disease
- Type A personality (behavior pattern) includes competitiveness,
time urgeny, and hostilityQ.
- Type D (distressed) personality is not only a risk factor for
development of ischemic heart disease, it also leads to worse
prognosis following a myocardial infarctionQ.
Type - A Personality
- It is originally characterized by competitiveness, time urgency,
hostility and angerQ (last two components are
pathogenic). Ambitiousness, aggressiveness, competitiveness,
impatience, muscle tenseness, alertness, rapid and emphatic vocal
style, irritation, and cynicism are other features.
- Are hard working workaholics who deny emotional or physical
vulnerability. Their self esteem is dependent on achievement i.e.
they are impatient, achievement oriented people.
- These individuals show a greater physiological reactivity (eg B P,
heart rate) and are more likely to get engaged in unhealthy
habbits eg, drinking alcohol, smoking. And if these individuals smoke
they inhale smoke for a longer time. Associated with significantly
high risk of coronary heart disease.
Type B Personality
It includes carefree, easy going, relaxed individuals
Type D Personality
- It includes people with distressed personality who suffer from high
degree of emotional distress (negative emotions/affectivity) but they
consciously suppress their feelings (=social inhibition).
Type D personality is both a prognostic and risk factor for
IHDQ. These persons have worse prognosis after MI with an - 4
times increased risk of sudden cardiac death, recurrent MI &
mortality.
- These persons have an impaired/poor quality of life and are more
likely to experience anxiety & depression (mood swings)Q.
* Risk factor for cardiac disase include major depressive disorder,
type D personality and type A personalityQ (esp anger & hostality)
197. Which of the following is the characteristic feature of Bipolar disorder II?
a) Hypomania plus mania

b) Depression alone

c) Mania and depression

d) Hypomania and depression

Correct Answer - D
Bipolar disorder is a chronic mood disorder characterized by the presence of mania (bipolar
I disorder) or hypomania and depression (bipolar II disorder). Manic episodes are
distinct periods of abnormally and persistent moods that can be euphoric, expansive, or
irritable.

Ref: Textbook of Family Medicine: Expert Consult - Online and Print By Robert E Rakel,
M.D., David Rakel, 2011, Page 1064.

Join Telegram Channel @mmedicalbooks for all NEET


PG /JIPMER/PGI/FMGE /AIIMS Questions papers
withanswers and explainations and All kinds of
TEXTBOOKS & NOTES Free Available in the Channel t.me/
mmedicalbooks or click on red box for the channel link
198. Which of the following is not a specific
somatoform disorders?.
a) Somatization disorder

b) Fibromyalgia

c) Chronic fatigue syndrome

d) Irritable bowel syndrome

Correct Answer - B
Ans--b. Fibromyalgia (Ref Harrison 19/e p2238. 18/e p122. 2849:
Kaplan and Sada'ock!s Synopsis of Psychiatry 10/e p276; Niraj
Ahuja 7/e p104-105)
Fibromyalgia is not a specific somatoform disorder, it has organic
basis behind its pathology
somatic like is a generic term first used in DSM III for a group of
disorders charactrized by physical symptoms that are not explained
by organic factors (i.e. for syndromes of presumed psychological
origin)."

Join Telegram Channel @mmedicalbooks for all NEET PG /


JIPMER/PGI/FMGE /AIIMS Questions papers withanswers
and explainations and All kinds of TEXTBOOKS & NOTES
Free Available in the Channel t.me/mmedicalbooks or click
on red box for the channel link
199. A 65-year-old man with diabetes, on an oral
hypoglycemic, presents to the ER with a sports- related
right shoulder injury. His heart rate was noted to be
irregular and the following ECG was obtained. The best
immediate therapy is

a) Atropine

b) Observation

c) Pacemaker

d) Electrical cardioversion

Correct Answer - B
This ECG shows Mobitz type I second-degree AV block, also known
as Wenckebach phenomenon, characterized by progressive PR
interval prolongation prior to block of an atrial impulse. This rhythm
generally does not require therapy. It may be seen in normal
individuals; other causes include inferior MI and drug intoxications
such as from digoxin, beta blockers, or calcium channel blockers.
Even in the post-MI setting, it is usually stable, although it has the
potential to progress to higher-degree AV block with consequent
need for pacemaker.

You might also like